Você está na página 1de 127

Name : ………………………………………………….

Class : ………………………………………………….

ADDITIONAL MATHEMATICS
PAPER 2
FOR SPM 2019 / 2010

FORM TOPICS PAGE REMARK


4 Chapter 4 Simultaneous Equations 3 – 11
4 Chapter 2 Quadratic Equations 12 – 15
4 Chapter 3 Quadratic Functions 16 – 20
4 Chapter 1 Functions 21 – 24
5 Chapter 1 Progressions 25 – 32
4 Chapter 7 Statistics 33 – 37
4 Chapter 5 Indices And Logarithms 38 – 42
4 Chapter 6 Coordinate Geometry 43 – 46
4 Chapter 9 Differentiation 47 – 52
5 Chapter 4 Vectors 53 – 57
5 Chapter 5 Trigonometric Functions 58 – 66
4 Chapter 8 Circular Measures 67 – 78
5 Chapter 2 Linear Law 79 – 89
5 Chapter 8 Probability Distributions 90 – 97
5 Chapter 10 Linear Programming 98 – 108
4 Chapter 11 Index Number 109 – 118
4 Chapter 10 Solution Of Triangles 119 – 127

FORMULAE

ALGEBRA
1. x = 8. loga b =
2. am × an = a m + n 9. Tn = a + (n – 1)d
3. am ÷ an = a m – n 10. Sn = [2a + (n – 1)d]
4. (am)n = amn 11. Tn = arn – 1
5. loga mn = loga m + loga n 12. Sn = = ,r1
6. loga = loga m – loga n 13. S = , r < 1
n
7. loga m = n loga m
CALCULUS
1. y= uv, =u +v 4. Area under a curve, =

2. y = , = =
3. = × 5. Volume of revolution =
=

BILINGUAL
DWIBAHASA
Prepared By: WongYH
SMK Merbau Miri
1
STATISTICS

1. = 6. I= × 100

2. = 7. =

3.  = 8. n
Pr =

n
= 9. Pr =

4.  = 10. P(A  B) = P(A) + P(B) – P(AB)

= 11. P(X = r) = nCr pr q n – r p+q=1

5. m = L + ( )c 12. Mean,  = np
13.  =
14. Z =

GEOMETRY
1. Distance =
2. Midpoint, (x, y) = ( , )
3. A point dividing a segment of a line
(x, y) = ( , )
4. Area of triangle
= (x1y2 + x2y3 + x3y1) – (x2y1 + x3y2 + x1y3)
5. r =
6. =

TRIGONOMETRY
1. Arc length, s = r θ
2. Area of sector, A = r2 θ
3. sin2 A + cos2 A = 1
4. sec2 A = 1 + tan2 A
5. cosec2 A = 1 + cot2 A
6. sin 2A = 2 sin A cos A
7. cos 2A = cos2 A – sin2 A
= 2 cos2 A – 1
= 1 – 2 sin2 A
8. sin (A  B) = sin A cos B  cos A sin B
9. cos (A  B) = cos A cos B sin A sin B
10. tan (A  B) =
11. tan 2A =
12. = =
2 2 2
13. a = b + c – 2 b c cos A
14. Area of triangle = a b sin C

2
FORM 4 CHAPTER 4 SIMULTANEOUS EQUATION
PAPER 2 SECTION A
1 Solve the simultaneous equations.
Selesaikan persamaan serentak berikut.
2x + y = 1 and x2 + y2 + xy = 7
Solution
Remember…..try to avoid fraction AND can be factorised as the question does not mention the answer in decimal place

Linear: 2x + y = 1
y = 1 – 2x ----------------(1)
Not linear: x2 + y2 + xy = 7 ----------------(2)
Substitute (1) into (2):
x2 + (1 – 2x)2 + x (1 – 2x) = 7 (1 – 2x)2 = 1 + 4x2 [Wrong] ← Don’t do this
2
x + (1 – 2x)(1 – 2x) + x (1 – 2x) = 7
x2 + 1 – 2x – 2x + 4x2 + x – 2x2 – 7 = 0 (1 – 2x)2 = (1 – 2x)(1 – 2x) [Correct]
3x2 – 3x – 6 = 0
x2 – x – 2 = 0 (1 – 2x)2 = 12 + (1)( –2)(2)x + (–2x)2
(x + 1)(x – 2) = 0
(x + 1) = 0 or (x – 2) = 0 = 1 – 4x + 4x2
x=–1 x=2

From (1), y = 1 – 2x
When x = – 1, y = 1 – 2(–1) When x = 2, y = 1 – 2(2)
=3 =–3

So, x = – 1 when y = 3 and x = 2 when y = – 3

2 Solve the following simultaneous equations. Give your answers correct to three decimal places.
Selesaikan persamaan serentak berikut. Berikan jawapan anda tepat kepada 3 tempat perpuluhan.
x2 + 4y2 = 37 and x – 2y = 8
Solution
Remember…..try to avoid fraction AND cannot be factorised as the question mentions the answer in decimal places

Linear: x – 2y = 8
x = 8 + 2y ----------------(1)
Not linear: x2 + 4y2 = 37 -----------------(2)
Substitute (1) into (2):
(8 + 2y)2 + 4y2 = 37
(8 + 2y)(8 + 2y) + 4y2 = 37 (8 + 2y)2 = 64 + 4y2 [Wrong] ← Don’t do this
64 + 16y + 16y + 4y2 + 4y2 = 37
64 + 16y + 16y + 4y2 + 4y2 – 37 = 0 (8 + 2y)2 = (8 + 2y)(8 + 2y) [Correct]
8y2 + 32y + 27 = 0
↑ ↑ ↑ (8 + 2y)2 = 82 + (8)(2)(2)y + (2y)2
a = 8 b = 32 c = 27
= 64 + 32y + 4y2
 b  b 2  4ac
From formula, y =
2a
 32  32 2  4(8)( 27)
= ←Must show this step
2(8)
= – 1.209 or – 2.791

From (1), x = 8 + 2y
When y = – 1.209 , x = 8 + 2(– 1.209) When y = – 1.209 , x = 8 + 2(– 2.791)
= 5.582 = 2.418

So, x = 5.582 when y = – 1.209 and x = 2.418 when y = – 1.209

3
EXERCISE
1 Solve the simultaneous equations [5 marks] [Ans: x = 2, y = – 4; x = 3, y = – 2]
Selesaikan persamaan serentak
y – 2x = – 8 and x2 – 3x – y = 2
Answer space

2 Solve the simultaneous equations [5 marks] [Ans: k = 3.624, h = 0.624; k = -6.624, h = -9.624]
Selesaikan persamaan serentak
k – h = 3 and k2 + 3h = 15
Give your answers correct to three decimal places.
Berikan jawapan anda tepat kepada tiga tempat perpuluhan.
Answer space

4
3 Solve the simultaneous equations [5 marks] [Ans: x= 8/3, y =11/3; x = 3, y =2]
Selesaikan persamaan serentak
y + 5x = 17 and y2 + 5x2 = 49
Answer space

4 Solve the simultaneous equations [5 marks] [Ans: x = 1.120, y = -1.080; x = -1.786, y = 0.857]
Selesaikan persamaan serentak
2x + 3y + 1 = 0 and x2 + 6xy + 6 = 0
Give your answers correct to three decimal places.
Berikan jawapan anda tepat kepada tiga tempat perpuluhan.
Answer space

5
5 The straight line y – x = 3 intersects the curve x2 + 2xy – 3y – 2x – 5 = 0 at two points. Find the coordinates of the points.
Garis lurus y – x = 3 menyilangi lengkung x2 + 2xy – 3y – 2x – 5 = 0 pada dua titik. Cari koordinat titik-titik tersebut.
[6 marks] [Ans: (-7/3, 2/3); (2, 5)]
Answer space

6 In Diagram 1, PQRS is a piece of manila card with an area of 588 cm2. A semicircular PAS is cut out from the manila
card. The perimeter of the remaining manila card is 186 cm. find the integer value of x and y.
Dalam Rajah 1, PQRS ialah sekeping kad manila dengan luas 588 cm2. Satu semibulatan PAS dipotong keluar daripada kad itu.
Perimeter bagi kad manila yang tertinggal ialah 186 cm. Cari nilai integer x dan y. [5 marks] [Ans: x = 6, y = 1]
P 14x cm Q

A 7y cm

S R
Diagram 1 / Rajah 1
Answer space

6
INTENSIVE EXERCISE
1 SPM 2018 P2 Q4
Diagram 1 shows the plan of a rectangular garden PQRS. The garden consists of a semicircular pond PTS
and grassy area PQRST.
Rajah 1 menunjukkan pelan bagi suatu taman segiempat PQRS. Taman itu terdiri daripada semibulatan kolam PTS
dan kawasan berumput PQRST.

Diagram 1 / Rajah 1
It is given that SR = 6y metre and QR = 7x metre, x  y. The area of the rectangular garden PQRS is 168
metre2 and the perimeter of the grassy area is 60 metre. The pond with uniform depth contains 15.4 metre 2 of
water. By using π = , find the depth, in metre, of the water in the pond. [7 marks / markah]
Diberi bahawa SR = 6y meter dan QR = 7x meter, x  y. Luas taman segiempat PQRS itu ialah 168 meter2 dan
perimeter kawasan berumput itu ialah 60 meter. Kolam dengan kedalaman seragam mengandungi 15.4 meter2 air.
Answer Space

2 SPM 2017 P2 Q1
Solve the following simultaneous equations:
Selesaikan persamaan serentak berikut:
x – 3y = 1 , x2 + 3xy + 9y2 = 7 [5 marks / markah] [Ans: x = 2, y = 1/3, x = -1, y = -2/3]
Answer Space

7
3 SPM 2016 P2 Q3
Adam planted vegetables on a piece of land. The shape of the land is a right angled triangle. Given the
longest side of the land is y metre. The other two sides of the land are x metre and (2x – 1) metre
respectively. He fenced the land with 40 metre of barbed wire.
Find the length, in metre, of each side of the land. [7 marks / markah] [Ans: 8, 15, 17]
Adam menanam sayur-sayuran pada sebidang tanah. Bentuk tanah ialah segitiga bersudut tegak. Diberi panjang sisi
terpanjang bagi tanah itu ialah y meter. Dua sisi tanah yang lain ialah x meter dan (2x – 1) meter masing-masing. Dia
memagari tanah itu dengan 40 meter dawai berduri.
Cari panjang, dalam meter, bagi setiap sisi tanah itu.
Answer Space

4 SPM 2015 P2 Q1
Solve the following simultaneous equations:
Selesaikan persamaan serentak berikut:
3x – y – 4 = 0 , 5x2 + 2y2 – 6xy = 16 [5 marks / markah] [Ans: x = 4/5, y = -8/5, x = 4, y = 8]
Answer Space

8
5 SPM 2014 P2 Q1
Solve the following simultaneous equations:
Selesaikan persamaan serentak berikut:
y – 2x + 1 = 0 , x2 – 2y2 – 3y + 2 = 0
Give your answer correct to three decimal places.
Beri jawapan anda betul kepada tiga tempat perpuluhan.
Answer Space [5 marks / markah][x = 0.813, y = 0.626, x = -0.527, y = -2.054]

6 SPM 2012 P2 Q1
Solve the simultaneous equations: y – 2x + 1 = 0 and 4x2 + 3y2 – 2xy = 7.
Selesaikan persamaan serentak:
Give the answers correct to three decimal places.
Beri jawapan betul kepada tiga tempat perpuluhan.
Answer Space [5 marks / markah][x = 1.129, y = 1.258, x = -0.295, y = -1.590]

9
7 P is a two digit number. When the two digits are reversed, a new number, Q, is obtained. The product of P
and Q is 574 and their difference is 27. Find the two numbers of P and Q.
P ialah satu nombor dengna dua digit. Apabila dua digit itu diterbalikkan, satu nombor baru, Q, diperolehi. Hasil
darab P dan Q ialah 574 dan perbezaannya ialah 27. Cari dua nombor P dan Q itu.
Answer Space [7 marks / markah][14, 41]

8 A piece of wire, 40 cm long, is cut into two parts which are then bent to form two squares as shown in the
diagram below.
Seutas dawai, 40 cm panjang, dipotong kepada dua bahagian di mana ianya dibengkok untuk membentuk dua
segiempat sama sisi seperti ditunjukkan dalam rajah bawah.

The total area of the two squares is 58 cm2. Find the lengths of the sides of the two squares.
Jumlah luas bagi kedua-dua segiempat ialah 58 cm2. Cari panjang setiap sisi segiempat sama sisi itu.
Answer Space [7 marks / markah][7, 3]

10
9 Find the values of p if the straight line 4x + y − p = 0 is a tangent to the curve x 2 = 3 – xy.
Cari nilai-nilai bagi p jika garis lurus 4x + y − p = 0 ialah tangen kepada lengkungan x 2 = 3 – xy.
Answer Space [6 marks / markah][Ans: p = ± 6]

10 In the diagram below, PQRS is a rectangular sheet of paper with an area of 224 cm2. QAR which is semicircular in
shape is cut out from the paper. The perimeter of the remaining paper is 72 cm.
Dalam rajah bawah, PQRS ialah satu segiempat tepat bagi sekeping kertas dengan luas 224 cm2. QAR ialah satu
semibulatan yang dipotong keluar daripada kertas itu. Perimeter bagi kertas yang tinggal ialah 72 cm.

Find the values of x and y. [Use / Guna π = ]


Cari nilai bagi x da y. [Guna π = ]
Answer Space [7 marks / markah][Ans: x = 24 , y = or x = 12, y = ]

11
FORM 4 CHAPTER 2 QUADRATIC EQUATIONS
PAPER 2 SECTION A
1 (a) The quadratic equation 3x2 + kx + h = 0, where k and h are constants, has roots 1
3
and 2. Find the value of k and
of h.
Persamaan 3x2 + kx + h = 0, dengan keadaan k dan h adalah pemalar, mempunyai punca-punca 1
3
and 2. Cari nilai k dan
nilai h.
Solution
The quadratic equation given in question must be arranged to the general form: ax2 + bx + c = 0
b c
Recall the formula: SOR = – POR =
a a

From 3x2 + kx + h = 0 ←Already in general form


We know that a = 3, b = k, c = h

Given the roots are 1


3
and 2

b c
SOR = – POR =
a a
k h
1
3
+ (2) = – 1
3
× (2) =
3 3
5 = – k 2 = h
3 3
3 3
k
= 53 h=–2
3
k=5
1 (b) Given that 4 and k are a roots of the quadratic equation x2 – 5x + p + 7 = 0, find the value of k and of p.
Diberi bahawa 4 dan k adalah punca-punca bagi persamaan kuadratik x2 – 5x + p + 7 = 0, cari nilai k dan nilai p.
Solution
From the quadratic equation x2 – 5x + p + 7 = 0 ← is already in general form
We know that a = 1, b = – 5 and c = p + 7
Given the roots are 4 and k
b c
SOR = – POR =
a a
5 p7
4+k=– 4×k=
1 1
4+k=5 4k = p + 7
k=1 4(1) = p + 7
4–7=p
p=–3
1 (c) Solve the quadratic equation 5(2x – 1) = (3x + 1)(x – 3). Give your answer correct to four significant figures.
Selesaikan persamaan kuadratik 5(2x – 1) = (3x + 1)(x – 3). Berikan jawapan anda betul kepada empat angka bererti.
Solution
Expand the 5(2x – 1) = (3x + 1)(x – 3) to general form
10x – 5 = 3x2 – 9x + x – 3
2
3x – 9x + x – 3 = 10x – 5
3x2 – 9x + x – 3 – 10x + 5 = 0
3x2 – 18x + 2 = 0
So, a = 3, b = – 18, c = 2

 b  b 2  4ac
From formula, x =
2a
 (18)  (18) 2  4(3)( 2)
x=
2(3)
x = 5.8868 or – 0.1132

12
1 (d) The quadratic equation 2x(x + 3) = 3 – 2k , where k is a constant, has two equal roots. Find the value of k.
Persamaan kuadratik 2x(x + 3) = 3 – 2k , dengan keadaan k ialah pemalar, mempunyai dua punca yang sama. Cari nilai k.
Solution
The 2x(x + 3) = 3 – 2k must be arranged to general form!
2x2 + 6x = 3 – 2k
2
2x + 6x – 3 + 2k = 0
So, a = 2, b = 6 and c = – 3 + 2k
Remember the formulae:
Two equal roots OR Intersect at one point means b2 – 4ac = 0
Two distinct roots OR Two different roots means b2 – 4ac > 0
No root OR No solution OR Does not intersect means b2 – 4ac < 0
Tangent to the curve means Solve by simultaneous equation like Chapter 4 Form 4
Then, b2 – 4ac = 0
(6)2 – 4(2)(– 3 + 2k) = 0
36 – 8(– 3 + 2k) = 0
36 – (– 24 + 16k) = 0
36 + 24 – 16k = 0
60 – 16k = 0
– 16k = – 60
16k = 60
60
k = 16
k= 15
4

1 (e) The straight line y = m – 2x does not intersect the curve y2 + xy + 8 = 0. Find the range of values of m.
Garis lurus y = m – 2x tidak bersilang dengan lengkung y2 + xy + 8 = 0. Carikan julat nilai m.
Solution
Let y = m – 2x ------------(1) and y2 + xy + 8 = 0 ------------(2)
Substitute (1) into (2):
(m – 2x)2 + x (m – 2x) + 8 = 0
(m – 2x) (m – 2x) + x (m – 2x) + 8 = 0
m2 – 2mx – 2mx + 4x2 + mx – 2x2 + 8 = 0
2x2 – 3mx + m2 + 8 = 0 ← to general form
So, a = 2, b = – 3m and c = m2 + 8
Does not intersect means b2 – 4ac < 0
(– 3m) – 4(2)(m2 + 8) < 0
2

9m2 – 8(m2 + 8) < 0


9m2 – (8m2 + 64) < 0
9m2 – 8m2 – 64 < 0
m2 – 64 < 0
m2 < 64
m < 8 or m < – 8
So, the range is – 8 < m < 8
1 (f) Given α and  are the roots of the quadratic equations 2x2 + 8x + 5 = 0.
Form the quadratic equation which has the roots 2α and 2.
Diberi α dan  ialah punca persamaan kuadratik 2x2 + 8x + 5 = 0.
Bentukkan persamaan kuadratik yang mempunyai punca 2α dan 2.
Solution
From 2x2 + 8x + 5 = 0 ← Remember must in general form!
So, a = 2, b = 8 and c = 5 with the roots are α and .
b c
Then, SOR = – and POR =
a a
α +  = – 8/2 and α ×  = 5/2
α+=–4 α  = 5/2
Form the quadratic equation using 2α and 2
New SOR = 2α + 2 and new POR = 2α × 2
= 2(α + ) =4α
= 2(– 4) = 4 (5/2)
=–8 =5
So, the new equation is in the form of x2 – (SOR) x + (POR) = 0  Remember this formula
x2 – (– 8) x + (5) = 0
x2 + 8x + 5 = 0
13
EXERCISE
1 The quadratic equation hx2 + kx + 3 = 0, where h and k are constants has two equal roots. Express h in term of k.
Persaman kuadratik hx2 + kx + 3 = 0, dengan keadaan h dan k adalah pemalar, mempunyai dua punca yang sama.
Ungkapkan h dalam sebutan k.
[2 marks] [Ans: h = k2/12]
Answer space

2 The quadratic equation 2x2 = px2 – 4(x + 2), where p is a constant, has no real roots. Find the range of the values of p.
Persamaan kuadratik 2x2 = px2 – 4(x + 2), dengan keadaan p ialah pemalar, tidak mempunyai punca nyata. Cari julat nilai p.
[3 marks] [Ans: p < 3/2]
Answer space

3 Find the values of n for which the curve y = n + 8x – x2 intersect the straight line y = 3 at one point. [4 marks] [Ans: -13]
Cari nilai n di mana lengkung y = n + 8x – x2 menyilang garis lurus y = 3 pada satu titik.
Answer space

14
4 Given one of the roots of a quadratic equation x2 + px + 4 = 0, where p is a constant, is four times the other root.
Diberi satu daripada punca persamaan kuadratik x2 + px + 4 = 0, dengan keadaan p ialah pemalar, adalah empat kali
punca yang satu lagi.
(a) Find the value of p if the roots are positive. [4 marks] [Ans: p = -5]
Cari nilai p jika punca-puncanya bernilai positif.
(b) Hence, form the quadratic equation which has the roots (p – 3) and 1
2
p. [3 marks] [Ans: 2x2 + 21x + 40 = 0]

Seterusnya, bentukkan persamaan kuadratik yang mempunyai punca (p – 3) dan 1


2
p.
Answer space

5 A quadratic equation x2 + 4(3x + m) = 0, where k is a constant has roots p and 2p, p  0.


Persamaan kuadratik x2 + 4(3x + m) = 0, dengan keadaan k ialah pemalar mempunyai punca-punca p and 2p, p  0.
(a) Find the value of p and of m. [4 marks] [Ans: p = – 4, m = 8]
Cari nilai p dan nilai m.
(b) Hence, form the quadratic equation which has roots m + 1 and m – 6. [3 marks] [Ans: x2 – 11x + 18 = 0]
Seterusnya, bentukkan persamaan kuadratik dengan punca-punca m + 1 dan m – 6.
Answer space

15
FORM 4 CHAPTER 3 QUADRATIC FUNCTIONS
PAPER 2 SECTION A

If given the function in the form of f (x) = a(x + b)2 + c


a > 0 → minimum point
a < 0 → maximum point
The maximum / minimum point is (–b, c) The maximum / minimum value is c
Axis of symmetry is x = – b
1 By expressing the function f (x) = 3x2 – 6x + 5 in form of f (x) = a(x – p)2 + q, find the minimum value of f (x).
Dengan mengungkapkan fungsi f (x) = 3x2 – 6x + 5 dalam bentuk f (x) = a(x – p)2 + q, cari nilai minimum bagi f (x).
Solution
For maximum or minimum point, do the completing the square f (x) = a(x + 2ba )2 + c – ( 2ba )2

From f (x) = 3x2 – 6x + 5 ← remember must be in general form


Taking out the 3:
f (x) = 3[ x2 – 2x + 5]
3
= 3[ x2 – 2x + (  2 )2 + 5 – ( 22 )2 ]
2 3
= 3[ x2 – 2x + (– 1)2 + 5
3
– (– 1)2 ]
= 3[(x – 1)2 + 5
3
– (– 1)2 ]
= 3[(x – 1)2 + ] 2
3
= 3(x – 1)2 + 2
So, a = 3, p = 1 and q = 2. The minimum value of f (x) is 2.
2 Find the maximum or minimum value for the function f (x) = 1 + 3x – 2x2. Thus, find the equation of the axis of
symmetry for the function.
Cari nilai maksimum atau minimum bagi fungsi f (x)= 1 + 3x – 2x2. Seterusnya, carikan persamaan paksi simetri bagi graf fungsi itu.
Solution
From f (x) = 1 + 3x – 2x2

To general form: f (x) = –2x2 + 3x + 1

Taking out – 2: f (x) = –2[x2 – 3


2
x – 1
2
]
↑ ↑
Add in +[ – 2 ÷2]
3 – [ – 32 ÷2]

f (x) = –2[ x2 – 32 x + ( 43 )2 – 12 – ( 43 )2 ]


-------------------- ---------------

= –2[(x – 34 )2 – 12 – ( 43 )2 ]
---------- ----------------

= –2[(x – 3
4
)2 – 17
16
]

= –2(x – 3
4
)2 + 17
8
]

17 3
The maximum value is 8
. The axis of symmetry is x = 4
. Extra info: Maximum point is ( 34 , 17
8
)

EXERCISE
1 Given the quadratic function y = – 2(x + 3) + 9. State 2

Diberi fungsi kuadratik y = – 2(x + 3)2 + 9. Nyatakan


(a) the coordinates of the maximum point, [1 mark] [Ans: (– 3, 9)]
koordinat titik maksimum,
(b) the equation of the axis of symmetry. [1 mark] [Ans: x = – 3]
persamaan paksi simetri.
Answer space

16
2 Diagram 2 shows the graph of the function y = – (x + 1)2 + 9, where m is a constant. The curve touches the line y = m at
point A and cut the y-axis at point B. The curve also cut the x–axis at point P.
Rajah 2 menunjukkan graf fungsi y = – (x + 1)2 + 9, dengan keadaan m ialah pemalar. Lengkung itu menyentuh garis y = m di titik A
dan menyilang paksi-y di titik B. Lengkung itu juga menyilang paksi-x di titik P.
y

A
y=m

B (0, k)
P
x
O
Diagram 2 / Rajah 2
(a) Determine the value of m and of k. / Tentukan nilai m dan nilai k. [2 marks] [Ans: 9, 8]
(b) State the coordinates of point P. / Nyatakan koordinat bagi titik P. [2 marks] [Ans: (-4,0)]
Answer space

3 Diagram 3 shows the graph of a quadratic function f (x) = 3(x + p)2 + 4, where p is a constant.
Rajah 3 menunjukkan graf fungsi kuadratik f (x) = 3(x + p)2 + 4, dengan keadaan p ialah pemalar.
y

y = f (x)

(– 2, q)
x
O

Diagram 3 / Rajah 3
The curve y = f (x) has the minimum point (2, q), where q is a constant.
Lengkung y = f (x) mempunyai titik minimum (2, q), dengan keadaan q adalah pemalar.
State / Nyatakan
(a) the value of p / nilai p [1 mark] [Ans: 2]
(b) the value of q / nilai q [1 mark] [Ans: 4]
(c) the equation of the axis of symmetry / persamaan paksi simetri. [1 mark] [Ans: x = – 2]
Answer space

17
4 Diagram 4 shows the graph of a quadratic function y = f (x). The straight line y = – 9 is a tangent to the curve y = f (x).
Rajah 4 menunjukkan graf fungsi kuadratik y = f (x). Garis lurus y = – 9 ialah tangen pada lengkung y = f (x).
y
y = f (x)

x
0 1 7

y=–9
Diagram 4 / Rajah 4
(a) Write the equation of the axis of symmetry of the curve. [1 mark] [Ans: x = 4]
Tuliskan persamaan paksi simetri bagi lengkung itu.
(b) Express f (x) in the form of (x + p)2 + q, where p and q are constants. [2 marks] [Ans: -4, -9]
Ungkapkan f (x) dalam bentuk (x + p)2 + q, dengan keadaan p dan q adalah pemalar.
Answer space

5 Find the range of values of x for which (3x + 2)(x – 5) > x – 5 . [3 marks] [Ans: x < – 1/3, x > 5]
Cari julat nilai x bagi (3x + 2)(x – 5) > x – 5 .
Answer space

6 Find the range of values of x for which (3x + 1)(2x – 1) > 3 + x . [3 marks] [Ans: x < – 2/3, x >1]
Cari julat nilai x bagi (3x + 1)(2x – 1) > 3 + x .
Answer space

18
7 Given that the graph of quadratic function f (x) = 2x2 + bx + 8 always lies above the x-axis. Find the range of values of b.
Diberi graf fungsi kuadratik f (x) = 2x2 + bx + 8 sentiasa berada di atas paksi-x. Cari julat nilai b. [3 marks] [Ans: -8<x<8]
Answer space
Always lies above the x-axis → No solution and then use b2 – 4ac < 0

8 Given the quadratic function f (x) = 6x – 1 – 3x2.


Diberi fungsi kuadratik f (x) = 6x – 1 – 3x2.
(a) Express the quadratic function f (x) in the form k + m(x + n)2, where k, m and n are constants. Determine whether
f (x) has a maximum or minimum value and state the value.
Ungkapkan fungsi kuadratik f (x) dalam bentuk k + m(x + n)2, dengan keadaan k, m dan n ialah pemalar. Tentukan sama ada
fungsi f (x) mempunyai nilai maksimum atau minimum dan nyatakan nilainya. [2 marks] [Ans: f (x) = 2 – 3(x – 1)2; 2]
(b) Sketch the graph function of f (x). [3 marks]
Lakarkan graf fungsi f (x).
(c) Find the range of p such that the equation 6x – 1 – 3x2 = p has two distinct roots. [3 marks] [Ans: p < 2]
Carikan julat p supaya persamaan 6x – 1 – 3x2 = p mempunyai dua punca yang berbeza.
Answer space

19
9 The function f (x) = x2 – 4kx + 5k2 + 1 has a minimum value of r2 + 2k, where r and k are constants.
Fungsi f (x) = x2 – 4kx + 5k2 + 1 mempunyai nilai minimum r2 + 2k, dengan keadaan r dan k adalah pemalar.
(a) By using the method of completing the square, show that r = k – 1.
Dengan menggunakan kaedah melengkapkan kuasa dua, tunjukkan bahawa r = k – 1.
(b) Hence, of otherwise, find the values of r and k if the graph of the function is symmetrical about x = r2 – 1.
Seterusnya, atau dengan cara lain, cari nilai r dan nilai k jika graf bagi fungsi itu bersimetri pada x = r2 – 1.
Answer space

10 Given the quadratic function f (x) = 4 – 3x – x2.


Diberi fungsi kuadratik f (x) = 4 – 3x – x2.
(a) Find the coordinates of the maximum point. [2 marks] [Ans: (-3/2, 25/4)]
Carikan koordinat bagi titik maksimumnya.
(b) Sketch the graph of f (x) for domain – 4  x  2. [2 marks]
Lakarkan graf f (x) itu untuk domain – 4  x  2.
(c) State the range to f(x). [1 marks] [Ans: -6 < f (x) < 25/5]
Nyatakan julat yang sepadan bagi f (x).
Answer space

20
FORM 4 CHAPTER 1 FUNCTIONS
PAPER 2 SECTION A
1 Diagram 1 shows the relation between set A and set B / Rajah 1 menunjukkan hubungan antara set A dan set B.
p 2

q 4
6
r
8
Set A Set B
Diagram 1 / Rajah 1
State / Nyatakan:
(a) Domain, codomain, object, image / Domain, kodomain, Objek, Imej
Solution Domain = {p, q, r} Codomain = {2, 4, 6, 8} Object = {p, q, r} Image = {2, 4, 6, 8}
(b) The range of the relation, / Julat hubungan itu,
Solution Range = {4, 8}
(c) Type of the relation / Jenis hubungan itu
Solution Many to one
2 Diagram 2 below shows function g where g (x) = m / Rajah 2 menunjukkan fungsi g di mana g (x) = m .
nx 2 nx 2
x g(x)
1 2
3 -2
Diagram 2 / Rajah 2
Find the value of m and n / Cari nilai bagi m dan n.
Solution
From g(x) = nxm2

When x = 1, g(x) = 2 When x = 3, g(x) = – 2


So, 2 = n(1m)2 So, – 2 = n(3m)2
2= m
n2
–2= m
3n  2

2(n – 2) = m –2(3n – 2) = m
m = 2n – 4 ---------(1) m = – 6n + 4 ------------(2)
(1) = (2) : 2n – 4 = – 6n + 4
2n + 6n = 4 + 4
8n = 8
n=1
From (1), m = 2n – 4
= 2(1) – 4
m=–2
3 Given the function f (x) = 2x + 1 and g(x) = 3 – kx , find / Diberi fungsi f (x) = 2x + 1 dan g(x) = 3 – kx , cari
(a) f (2)
Solution
f (2) means substitute 2 into the function f(x)
From f (x) = 2x + 1
f (2) = 2(2) + 1
=5
(b) the value of k such that g f (2) = – 7 / nilai bagi k dengan keadaan g f (2) = – 7.
Solution
From 5(a), f (2) = 5
Given g f (2) = – 7 and g(x) = 3 – kx
So, g[ f (2)] = – 7
g [5] = – 7
3 – k (5) = – 7
3 – 5k = – 7
– 5k = – 7 – 3
– 5k = – 10
5k = 10
k=2

21
4 Given the function f (x) = 4x + 3 and g (x) = x2 – 5x + 1 / Diberi bahawa f (x) = 4x + 3 dan g(x) = x2 – 5x + 1.
Find / Cari
(a) f –1
Solution
From f (x) = 4x + 3
For inverse function, let 4x + 3 = y and find x !!
4x = y – 3
y 3
x= 4
x3
So, f – 1 (x) =
4
(b) f -1 (7)
Solution
x3
From question 4(a), we know that f – 1 (x) =
4
7 3
Then, f – 1 (7) =
4
f – 1 (7) = 1
(c) g f (x)
Solution
g f (x) means substitute function f (x) into the function g (x) as shown below:

f (x) = 4x + 3 and g (x) = x2 – 5x + 1

So,

g f (x) = g [f (x)]
= (4x + 3)2 – 5(4x + 3) + 1
= (4x + 3)(4x + 3) – 5(4x + 3) + 1
= 16x2 + 12x + 12x + 9 – 20x – 15 + 1
= 16x2 + 4x – 5
5 Given the function f (x) = x + 1. Find the function g if fg(x) = x2 + 3x + 5
Diberi fungsi f (x) = x + 1. Cari fungsi g jika fg(x) = x2 + 3x + 5
Solution
From fg(x) = x2 + 3x + 5
f [g(x)] = x2 + 3x + 5
g(x) + 1 = x2 + 3x + 5
g(x) = x2 + 3x + 4

6 Given the function f (x) = x + 3. Find the function g if gf (x) = x2 + 6x + 7


Diberi fungsi f (x) = x + 3. Cari fungsi g jika fg(x) = x2 + 6x + 7
Solution
From gf (x) = x2 + 3x + 5
g [f (x)] = x2 + 3x + 5
g[x + 3] = x2 + 3x + 5 ----------------(1)

Let x + 3 = y
x=y–3

From (1), g[y] = (y – 3)2 + 3(y – 3) + 5


g(y) = (y – 3)(y – 3) + 3(y – 3) + 5
g(y) = y2 – 3y – 3y + 9 + 3y – 9 + 5
g(y) = y2 – 3y + 5

So, g(x) = x2 – 3x + 5

22
EXERCISE
1 Diagram 1 shows the relation between set A and set B.
Rajah 1 menunjukkan hubungan antara set A dan set B.
Set B

Set A
0 1 2 3 4
Diagram 5

(a) Represent the above relation in set of ordered pairs.


Wakilkan hubungan di atas dalam bentuk set hubungan bertertib.
Answer space

(b) State the images of 1.


Nyatakan imej bagi 1.
Answer space

(c) State the type of the relation.


Nyatakan jenis hubungan.
Answer space

(d) State the range of the relation.


Nyatakan julat bagi hubungan itu.
Answer space

2 xk
Diagram 2 shows the function g : x → , x  0 where k is a constant. [2 marks] [Ans: k = 1]
x
xk
Rajah 2 menunjukkan fungsi g : x → , x  0 dengan keadaan k ialah pemalar.
x
x x+k
x
3
1
2
Diagram 2 / Rajah 2
Find the value of k / Cari nilai bagi k.
Answer space

23
3 x7
Given that the inverse function of f : x → 3x + m is f -1: x → , find the values of m and n. [3 marks][Ans:m= – 7, n= 3]
n
x7
Diberi fungsi songsang bagi f : x → 3x + m ialah f -1: x → , cari nilai m dan n.
n
Solution space

4 Given the function f : x → 3x – 4, find / Diberi fungsi f : x → 3x – 4, cari


(a) f –1(x) [1 mark] [Ans: (x + 4)/3]
Solution space

(b) the value of p such that f –1 (2p – 1) = p / nilai p dengan keadaan f –1 (2p – 1) = p [2 marks] [Ans: p = 3/2]
Solution space

5 Given the function g : x → 3x – 2 and g f : x → 3x2 + 4, find / Diberi fungsi g : x → 3x – 2 dan g f : x → 3x2 + 4, cari
(a) g –1 (x) [2 marks] [Ans: (x + 2)/3]
Solution space

(b) f (x) [2 marks] [Ans: x2 + 2]


Solution space

6 Given f : x → k – mx. Find / Diberi f : x → k – mx. Carikan


(a) f –1(x) in terms of k and m, / f –1(x) dalam sebutan k dan m [2 marks] [Ans: [k – x)/m]
Solution space

(b) The values of k and m if f –1(14) = – 4 and f (5) = – 13 [2 marks] [Ans: m = 3, k = 2]


Nilai-nilai k dan m jika f –1(14) = – 4 dan f (5) = – 13
Solution space

24
FORM 5 CHAPTER 1 PROGRESSION
PAPER 2 SECTION A
Arithmetic Progression Geometric Progression
1) Sebutan pertama, a 1) Sebutan pertama, a
2) Beza sepunya, d = T2 – T1 2) Nisbah sepunya, r =
3) The nth term, Tn = a + (n – 1)d
4) The last term, l = a + (n – 1)d 3) The nth term, Tn = ar n – 1
4) The last term, l = ar n – 1
5) Sum of first n terms, Sn = [2a + (n – 1)d]
5) Sum of first n terms, Sn = ,r>1
6) If given formula Sn, then Tn = Sn – Sn – 1
and T1 = S1 or Sn = ,r<1
6) Sum to infinity, S =
7) If given formula Sn, then Tn = Sn – Sn – 1
and T1 = S1
1. The third and eight terms of an arithmetic progression are – 5 and 15 respectively. Find
Sebutan ketiga dan kelapan bagi suatu janjang aritmetik ialah – 5 dan 15 masing-masing. Cari
(a) the first term and the common difference / sebutan pertama dan beza sepunya
(b) the sum of the first 10 terms / hasil tambah bagi 10 sebutan pertama [(a) a = -13, d = 4 (b) 50]
Answer Space

2. The first three terms of an arithmetic progression are 2k, 3k + 3 and 5k + 1. Find
Tiga sebutan pertama suatu janjang aritmetik ialah 2k, 3k + 3 dan 5k + 1. Cari
(a) the value of k, / nilai k,
(b) the sum of the first 15 terms of the progression [(a) k = 5 (b) 990]
hasil tambah bagi 15 sebutan yang pertama untuk janjang itu
Answer Space

3. 51, 58, 65,….191 are the first n terms of an arithmetic progression. Find the value of n.
51, 58, 65,….191 ialah n sebutan pertama bagi suatu janjang aritmetik. Cari nilai n. [Ans: 21]
Answer Space

25
4. Find the sum of all the multiples of 7 between 100 and 500. [17157]
Cari hasil tambah semua gandaan bagi 7 antara 100 dan 500.
Answer Space

5. The sum of the first 6 terms of an arithmetic progression is 39 and the sum of next 6 terms is – 69.
Find
Hasil tambah bagi 6 sebutan pertama suatu janjang aritmetik ialah 39 dan jumlah bagi 6 sebutan seterusnya ialah – 69.
Cari
(a) the first term and the common difference / sebutan pertama dan beza sepunya
(b) the sum of all the terms from the 15th term to the 25th term. [(a) a = 14, d = -3 (b) -473]
hasil tambah semua sebutan dari sebutan ke-15 hingga ke-25
Answer Space

n
6. Given the Sn = (2n + 1) represents the sum of the first n terms of the arithmetic progression.
2
Find
n
Diberi Sn = (2n + 1) mewakili hasil tambah bagi n sebutan pertama suatu janjang aritmetik. Cari
2
(a) first term / sebutan pertama
(b) second term / sebutan kedua
(c) sum of first 8 terms / hasil tambah 8 sebutan pertama
Answer Space

26
7. Given the first 3 terms in geometric progression: 18, 9, 4 ½ …….Find
Diberi 3 sebutan pertama dalam janjang geometri: 18, 9, 4 ½ …….Cari
(a) the common ratio / nisbah sepunya (c) the sum of first 6 terms / hasil tambah 6 sebutan pertama
(b) the 7th term / sebutan ke-7 (d) the sum to infinity / hasil tambah hingga ketakterhinggaan
Answer Space

8. Find the following recurring decimal as a fraction in its simplest form.


Cari nombor perpuluhan jadi semula sebagai pecahan dalam bentuk termudah bagi setiap berikut.
(a) 0.181818…… (b) 0.83333….. (c) 1.7777…..
Answer Space

9. Given the 2x + 3, x and x – 2 are the first three consecutive terms of a geometric progression with a
common ratio, r. Find
Diberi 2x + 3, x dan x – 2 ialah tiga sebutan berturutan dalam suatu janjang geometri dengan nisbah sepunya, r. Cari
(a) the value of x, / nilai x,
(b) the first term and value of common ratio, r / sebutan pertama dan nilai bagi nisbah sepunya, r
(c) the sum to infinity for the progression / hasil tambah hingga ketakterhinggaan bagi janjang itu
Answer Space

27
1 1
10. Given that = 0.16666….. / Diberi bahawa = 0.16666…..
p p
= 0.1 + a + b + …… = 0.1 + a + b + ……
Find the values of a and b. Hence, find the value of p.
Cari nilai bagi a dan b. Seterusnya, cari nilai bagi p.
Answer Space

11. Encik Ali and Encik Tan start to save money at the same time.
Encik Ali dan Encik Tan mula menyimpan duit pada masa yang sama .
(a) Encik Ali saves RMx in the first month and his savings increases constantly by RMy every
subsequent month. He saves RM200 in the 6th month and the total savings for 12 months is
RM2520. Find the values of x and y.
Encik Ali menyimpan RMx pada bulan pertama dan simpanannya bertambah secara tetap dengan RMy setiap
bulan seterusnya. Cari nilai x dan y.
(b) Encik Tan saves RM200 in the first month and his savings increases constantly by RM10 every
month. Find the value of n when both of them save the same amount of the money in nth
month.
Encik Tan menyimpan RM200 pada bulan pertama dan simpanannya bertambah secara tetap dengan RM10 setiap
bulan. Cari nilai n apabila kedua-dua mereka mempunyai simpanan duit yang sama dalam bulan ke-n.
Answer Space

28
12. Diagram shows the arrangement of equilateral triangles. The number of triangles in the lowest is 1.
For each of the other level, the number of triangles is 2 more than the level below it. The height of
each level is 3 cm. The highest level has 401 triangles.
Rajah menunjukkan susunan segitiga sama sisi. Bilangan segitiga di aras bawah ialah 1. Untuk setiap aras berikutnya,
bilangan segitiga ialah 2 lebih daripada aras bawah. Tinggi setiap aras ialah 3 cm. Aras tertinggi mempunyai 401
segitiga.

3 cm
Calculate / Hitung
(a) the height, in cm, to the highest level,
tinggi, dalam cm, ke aras tertinggi,
(b) the total number of triangles for the whole structure.
bilangan segitiga untuk seluruh struktur itu.
Answer Space

13. Diagram shows several trapeziums with areas A1, A2, A3, A4, A5 and so on. The height of the first
perpendicular edge is 2 cm and the height increases by 2 cm subsequently. The distance between the
two perpendicular edge of each trapezium is 2cm.
Rajah menunjukkan beberapa trapezium dengan luas A1, A2, A3, A4, A5 dan seterusnya. Tinggi untuk tepi serenjang yang
pertama ialah 2 cm dan tinggi itu bertambah sebanyak 2 cm seterusnya. Jarak antara dua sisi serenjang untuk setiap
trapezium ialah 2 cm.

10 cm
8 cm
6 cm
4 cm
2 cm A1 A2 A3 A4 A5
2 cm 2 cm 2 cm 2 cm
Find / Cari
(a) the height of the 25th perpendicular edge, / tinggi sisi serenjang yang ke-25,
(b) the sum of the areas for the first 10 trapeziums. / hasil tambah luas untuk 10 trapezium yang pertama
Answer Space

29
SPM DRILLING (KBAT)
1 The volume of water in a tank is 500 litres on the first day. The tank leaks at a rate of 15 litres per day. Find
the volume of water left at the end of the tenth day.
Isipadu air di dalam sebuah tangki ialah 500 liter pada hari pertama. Tangki itu bocor dan menitis dengan kadar 15 liter
sehari. Cari isipadu air yang tinggal pada akhir hari kesepuluh.
Answer Space

2 The price of an antique painting increases by 10% every year. If the price of the painting at the beginning of
the year 2000 was RM25000, what was its price at the beginning of the year 2010?
Harga sebuah lukisan antik bertambah sebanyak 10% setiap tahun. Jika harga lukisan itu pada awal tahun 2000 ialah
RM25000, berapakah harga lukisan itu pada awal tahun 2010?

30
3 Sunita has RM 250 in her piggy bank. Subsequently, she saves RM4 every day beginning from the 1st of
November. Calculate the total amount of money in her piggy bank at the end of November.
Sunita mempunyai RM250 dalam wang tabungnya. Kemudian dia menyimpan RM4 setiap hari bermula dari 1
November. Hitung jumlah wang dalam tabungnya pada akhir bulan November.
Answer Space

4 Given = 0.272727… is a recurring decimal. Find the value of k.


Diberi = 0.272727…ialah nombor perpuluhan jadi semula. Cari nilai k.
Answer Space

5 A ball is dropped from a height of 3 m. It rebounds to a height that is 4/5 of its previous height. The process
continues until the ball stops rebounding. Find the total distance travelled by the ball until it stops.
Sebiji bola dijatuhkan dari ketinggian 3 m. Bola itu melantun ke ketinggian iaitu 4/5 daripada ketinggian sebelumnya.
Proses ini berterusan sehingga bola itu berhenti. Cari jumlah jarak yang dilalui oleh bola itu sehingga ia berhenti.
Answer Space

31
6 A ball is dropped from a height of 80 m above a floor. Each time after the ball hits the floor, it will rebound
three-quarters of the distance fullen. Find
Sebiji bola dilepaskan dari ketinggian 80 m dari lantai. Setiap kali bola itu menghentam lantai, bola itu akan melantun
setinggi tiga perempat daripada jarak yang jatuh. Cari
(a) The height, in m, that the ball will reach when it rebound for the fifth time.
Tinggi, dalam m, yang dicapai oleh bola itu apabila melantun pada kali kelima.
(b) The total distance, in m, travelled when the ball hits the floor for the fifth time.
Jumlah jarak, dalam m, yang dilalui apabila bola itu menghentam lantai pada kali kelima.
(c) The total distance, in m, travelled before the ball comes to rest.
Jumlah jarak, dalam m, yang dilalui sebelum bola itu berhenti .
Answer Space

7 (a) The bacteria in a culture solution double every 5 minutes. Professor Lee begins an experiment with 20
bacteria at 10.00 am, find
Bakteria di dalam suatu larutan kultur berganda setiap 5 minit. Profesor Lee memulakan suatu eksperimen
dengan 20 bakteria pada pukul 10.00 am, cari
(i) The number of bacteria at 10.30 am
Bilangan bakteria pada pukul 10.30 am
(ii) The time when the number of bacteria is 10240
Masa apabila bilangan bakteria ialah 10240
(b) At 11.00 am, the bacteria stop growing and begin to die at a rate of 500 per minutes. Determine the
number of bacteria at 11.15 am.
Pada pukul 11.00 am, bakteria itu berhenti bertambah dan mula mati pada kadar 500 seminit. Tentukan bilangan
bakteria pada pukul 11.15 am.
Answer Space

32
FORM 4 CHAPTER 7 STATISTICS
PAPER 2 SECTION A
Remember the formulae [Refer to the formula page given]
Ungrouped data Grouped data
Mode → nombor repeated the most Mean, x =  xf

Mean, x = x f
N 1 N F
Median → data located at the centre Median, M = L + ( 2
)c

fm
x2
Variance, 2 = – ( x )2 1
4
N F
N First quartile, Q1 = L + ( )c
fm
Standard deviation,  =
x 2
 (x)
2 3
4
N F
N First quartile, Q3 = L + ( )c
fm
Effect on the range, interquartile, variance and standard
Variance,  2
=
x f 2
– ( x )2
f
deviation when each value in a set are changed
New mean +, – ,×,÷

New median +, – ,×,÷ Standard deviation,  =


 x f  (x)
2
2

New range ×,÷


f
New interquartile range ×,÷ L = Lower class boundary
F = cumulative frequency before the class
New variance ×,÷ [must square] f m =frequency of the class
c = size of class
New standard deviation × , ÷

1 A set of examination marks x1 , x2 , x3 , x4 , x5 , x6 , has a mean of 5 and a standard deviation of 1.5.


Satu set markah peperiksaan x1 , x2 , x3 , x4 , x5 , x6 , mempunyai min 5 dan sisihan piawainya ialah 1.5.
(a) Cari / find
(i) the sum of the marks,  x / jumlah markah  x
(ii) sum of the squares of the marks,  x
2

jumlah kuasa dua markah-markah tersebut,  x


2
.
(b) Each mark is multiplied by 2 and then 3 is added to it. Find, for this new set of marks,
Setiap markah didarab dengan 2 dan kemudian ditambah 3. Cari untuk set markah baru ini,
(i) Mean / min,
(ii) Variance / varians
Solution
(a)(i) Mean = 5 (a)(i) Standard deviation = 1.5
From x = x x 2
2
N From  =  (x)
x
N

x
5= 2
6
 (5) 2
 x = 30
1.5 =
6

1.52 =
 x2
– (5)2
6
 2
x = 163.5

(b) Each mark is multiplied by 2 and then 3 is added to it


(i) Mean = 5 × 2 + 3
= 13
(ii) Given the standard deviation = 1.5
New standard deviation = 1.5 × 2
=3
So, new variance = (3)2 Variance = (Standard deviation)2
=9

33
2 The table below shows the cumulative frequency distribution for the scores of 50 students in a competition.
Jadual di bawah menunjukkan taburan kekerapan longgokan bagi skor 50 orang murid dalam suatu pertandingan.
Score / Skor < 10 < 20 < 30 < 40 < 50
Number of students / Bilangan pelajar 5 19 37 44 50
(a) Based on the table above, copy and complete the following table.
Berdasarkan jadual di atas, salin dan lengkapkan Jadual.
Score / Skor 0–9 10 – 19 20 – 29 30 – 39 40 – 49
Number of students / Bilangan pelajar
Solution
Score / Skor 0–9 10 – 19 20 – 29 30 – 39 40 – 49
Number of students / Bilangan pelajar 5 14 18 7 6
(b) Without drawing an ogive, find the median and interquartile range of the distribution.
Tanpa melukis ogif, cari median dan julat antara kuartil bagi taburan itu.
Solution
Median First quartile, Q1 Third quartile, Q3 Interquartile range
= Observe at-( 12 × N)th = Observe at-( 14 × N)th = Observe at-( 34 × N)th = Q3 – Q1
= 30.2143 – 14.8571
= Observe at-( 12 × 50)th = Observe at-( 14 × 50)th = Observe at-( 34 × 50)th = 15.3572
= Observe at-25th = Observe at-12.5th = Observe at-37.5th
= In class 20 – 29 = In class 10 – 19 = In class 30 – 39
1 N F 1 N F 1 N F
=L+( 2 )c =L+( 4 )c =L+( 4 )c
fm fm fm
1
2
 50  19 1
4
 50  5 3
4
 50  37
= 19.5 + ( )10 = 9.5 + ( )10 = 29.5 + ( )10
18 14 7
= 22.83 = 14.8571 = 30.2143
3 The mean of a set of numbers 3, x, 6, 3x – 2, 9 and 12 is 8.
Min bagi suatu set nombor 3, x, 6, 3x – 2, 9 dan 12 ialah 8.
(a) Find / Cari
(i) The value of x, / nilai x
(ii) The variance, / varians
(b) Each number in the set is multiplied by 4 and then 10 is taken away from it. For this set of numbers, find
Setiap nombor dalam set itu didarabkan dengan 4 dan kemudiannya ditolak dengan 10. Bagi set nombor ini, cari
(i) The mean, / min
(ii) The standard deviation / sisihan piawai
Solution
(a) (i)
From mean, x = x (ii) Set data: 3, x, 6, 3x – 2, 9, 12 become 3, 5, 6, 13, 9, 12
N
3  x  6  3x  2  9  12 From variance, 2 =
x2
– ( x )2
8= N
6
4 x  28 32  5 2  6 2  132  9 2  12 2
8= = – (8)2
6 6
48 = 4x + 28 = 13.3333
48 – 28 = 4x
20 = 4x
x=5

(b) (i) New mean = Old mean × 4 – 10 (ii) From (a)(ii), variance = 13.3333
= 8 × 4 – 10 Then, standard deviation = 13.3333
= 22 = 3.6515
So, new standard deviation = Old standard deviation × 4
= 3.6515 × 4
= 14.606

34
EXERCISE
1 Table shows the marks obtained by 36 candidates in an examination.
Jadual di bawah menunjukkan markah yang diperolehi 36 orang calon dalam suatu peperiksaan.
Marks / Markah Number of candidates / Bilangan calon
40 – 49 4
50 – 59 5
60 – 69 6
70 – 79 9
80 – 89 8
90 – 99 4
(a) Without drawing an ogive, find the third quartile. [3 marks] [Ans: 83.25]
Tanpa melukis ogif, cari kuartil ketiga.
(b) Hitung / Calculate
(i) the mean, / min [2 marks] [Ans: 71.1667]
(ii) the standard deviation / sisihan piawai. [3 marks] [Ans: 15.0922]
Answer space

2 A set of eight numbers has a mean of 11.


Satu set lapan nombor mempunyai min 11.
(a) Cari / Find x. [1 mark] [Ans: 88]

(b) When a number k is added to this set, the new mean is 10. Find the value of k. [2 marks] [Ans: 2]
Apabila nombor k ditambah ke dalam set itu, min yang baru ialah 10. Cari nilai k.
Answer space

3 A set of 5 numbers, y1 , y 2 , y3 , y 4 and y5 , has a mean of 6 and a variance of 4.


For the set of data 3 y1  1, 3 y 2  1, 3 y3  1, 3 y 4  1 and 3y5 +1 , calculate
Satu set lima nombor, y1 , y 2 , y3 , y 4 dan y5 mempunyai min 6 dan varian ialah 4.
Bagi set data 3 y1  1, 3 y 2  1, 3 y3  1, 3 y 4  1 dan 3y5 + 1 ,hitung
(a) The mean / min [2 marks] [Ans: 19]
(b) Variance / varians [2 marks] [Ans: 36]
Answer space

35
4 The heights, in cm, of five players in a basketball team are 168, 170, 172, 175, 180.
Tinggi, dalam cm, lima pemain dalam pasukan bola keranjang ialah 168, 170, 172, 175, 180.
For the heights of the players, find
Untuk ketinggian pemain-pemain itu, cari
(a) mean / min [1 mark] [Ans: 173]
(b) the standard deviation / sisihan piawai. [3 marks] [Ans: 4.195]
Answer space

5 Diagram 5 is a histogram which represents the distribution of the marks obtained by 60 students in a test.
Rajah 5 menunjukkan histogram yang mewakili taburan markah yang diperoleh oleh 60 pelajar di dalam satu ujian.

20
18
16
Number of students

14
12
10
8
6
4
2
0
0.5 1 10.5 20.5 30.5 40.5 50.5 60.5
Marks

Diagram 5 / Rajah 5
(a) Without using an ogive, calculate the median mark. [3 marks] [Ans: 25.5]
Tanpa menggunakan ogif, hitung markah median.
(b) Calculate the standard deviation of the distribution. [4 marks] [Ans: 13.92]
Hitung sisihan piawai taburan itu.
Answer space

36
6 Table shows the total time spent on doing school homework by 120 students for a period of 4 weeks.
Jadual menunjukkan jumlah masa yang diluangkan untuk membuat kerja rumah sekolah oleh 120 pelajar dalam jangka masa 4minggu.
Total Time (hours) Number of students
Jumlah masa ( jam) Bilangan pelajar
5 – 14 12
15 – 24 17
25 – 34 26
35 – 44 31
45 – 54 16
55 – 64 10
65 – 74 8
Calculate / Hitung
(a) The mean, / min [2 marks] [Ans: 36.5]
(b) The interquartile range / julat antara kuartil [4 marks] [Ans: 22.12]
Answer space

7 A set of data consists of nine numbers. The sum of the number is 171 and the sum of the squares of the numbers is 3291.
Suatu set data mengandungi sembilan nombor. Hasil tambah nombor-nombor itu ialah 171 dan hasil tambah kuasa dua nombor-
nombor itu ialah 3291.
(a) For the nine number, find
Bagi sembilan nombor itu, cari
(i) the mean / min [1 mark] [Ans: 19]
(ii) the standard deviation / sisihan piawai [2 marks] [Ans: 2.16]
(b) If 6 is added to each number in the set, find
Jika 6 ditambahkan kepada setiap nombor di dalam set, cari
(i) the new mean / min baru [1 mark] [Ans: 25]
(ii) the new standard deviation / sisihan piawai baru [1 mark] [Ans: 2.16]
Answer space

37
FORM 4 CHAPTER 5 INDICES AND LOGARITHMS
PAPER 2 SECTION A
1 Solve / Selesaikan
1
8x  [2 marks] [Ans: -5/3]
32
Answer space
→ Find the smallest common factor for both 8 and 32.
→ Use the formula: am × an = a m + n OR am ÷ an = a m – n
→ Remember a0 = 1
→ Remember 1n = a – n
a

2 Solve the equation / Selesaikan persamaan [2 marks] [Ans: 13]


x 1 x 3
16 8
Answer space

3 Solve the equation / Selesaikan persamaan [3 marks] [Ans: -3/5]


3 x 3 1
27 
9 x 3
Answer space
→ Find the smallest common factor for both 27 and 9.
→ Use the formula: am × an = a m + n OR am ÷ an = a m – n
→ Remember 1n = a – n
a
→ Remember that means power of 1
2

38
4 Sove the equation / Selesaikan persamaan [4 marks] [Ans: 3]
22x + 1 – 13(2x) – 24 = 0
Answer space
→ Indices involving addition or subtraction: Move one part to the right side or straight using substitution method!!
22x + 1 – 13(2x) – 24 = 0
2 × 21 – 13(2x) – 24 = 0
2x

(2x )2 × 2 – 13(2x) – 24 = 0
Substitute 2x with y:
So, y2 × 2 – 13y – 24 = 0
2y2 – 13y – 24 = 0
(2y + 3)(y – 8) = 0
2y + 3 = 0 or y – 8 = 0
y=–¾ y=8

From 2x = y , When y = -3/4, When y = 8,


2x = -3/4 [rejected] 2x = 8
2x = 23
x=3
5 Sove the equation / Selesaikan persamaan [4 marks] [Ans: 3]
165x – 49x – 6 = 0
Answer space
→ Indices involving addition or subtraction: Move one part to the right side or straight using substitution method!!

6 Given that log3 K – log9 L = 2 , express K in terms of L. [4 marks] [Ans: K = 9 L]


Diberi log3 K – log9 L = 2, ungkapkan K dalam sebutan L
Answer space
→Rule 1: Logarithms involving addition or subtraction: Change to the same base and must combine together!
→Rule 2: loga m n = loga m + loga n
→Rule 3: loga mn = loga m – loga n
→Rule 4: log a m n = n loga m
→Rule 5: log a a = 1
logc b
→Rule 6: loga b = logc a
← only this one you will see the log divided by log
b
→Rule 7: If a = c, then loga c = b
Given log3 K – log9 L = 2 ←Under Rule 1
log 3 L
Change base! log3 K – =2 ←Under Rule 6: log9 L= log3 L
log 3 9 log 3 9
log 3 L
log3 K – =2
log 3 32
log 3 L
log3 K – =2 ←Under Rule 4: log334 = 4log33
2 log 3 3
log 3 L
log3 K – =2 ←Under Rule 5: log33 = 1
2

Multiply all terms by 2: 2log3 K – log3L = 4


log3 K 2 – log3L = 4 ←Under Rule 4: 2log3K = log3K2
K2 K2
log 3 =4 ←Under Rule 3: log3K2 – log3L = log 3
L L
K2
= 34 ←Under Rule 7
L
K2
= 81
L
2
K = 81L
K=9 L

39
7 (a) Given that logp3 = r and logp7 = s, express logp( 49 p ) in terms of r and s.
27
49 p
Diberi logp3 = r dan logp7 = s, ungkapkan logp( ) dalam sebutan r dan s.
27
Solution
If the log form given is in fraction, you must separate it into parts to solve!!!!

logp( 49 p ) = logp 49p – logp 27 ← Formula: loga m


n
= loga m – loga n
27
= logp (7 × 7 × p) – logp (3 × 3 × 3)
= logp 7 + logp 7 + logp p – [logp 3 + logp 3 + logp 3] ← Formula: loga m × n = loga m + loga n
= s + s + 1 – [r + r + r]
= 2s – 3r + 1
(b) Given that logx 2 = p and logx 7 = q, express logx ( 56
) in terms of p and q. [4 marks] [Ans: q + 3p – 2]
x2
56
Diberi logx 2 = p dan logx 7 = q, ungkapkan logx ( ) dalam sebutan p dan q.
x2
Answer space

(c) Given that log5 2 = q and log5 9 = p, express log5 8.1 in terms of q and p. [4 marks] [Ans: 2p – q – 1]
Diberi log5 2 = q dan log5 9 = p, ungkapkan log5 8.1dalam sebutan q dan p
Answer space
Change 8.1 to fraction number first! 8.1 = 81/10

8 (a) Solve the equation / Selesaikan persamaan


3 + log3 (2x – 1) = log3x
Solution
Change all to log3: log333 + log3 (2x – 1) = log3 x ← 3 = 3 × 1, 3 = 3 × log33, 3 = log333
log327 + log3 (2x – 1) = log3 x
log3[27 × (2x – 1)] = log3 x ← Formula: loga m × n = loga m + loga n
log3[27(2x – 1)] = log3 x
By comparing: 27(2x – 1) = x
54x – 27 = x
54x – x = 27
53x = 27
x = 27/53
(b) Given that log3 mn = 3 + 2log3 m – log3 n, express m in terms of n. [4 marks] [Ans: m = n2/27]
Diberi log3 mn = 3 + 2log3 m – log3 n, ungkapkan m dalam sebutan n.
Answer space

40
9 (a) Given that log9 y = log3 18, find the value of y. [3 marks] [Ans: y = 324]
Diberi log9 y = log3 18, cari nilai y.
Answer space
Hint: Change to same base first!!!

(b) Given that log3 m = v and log3 n = w, express log9 ( 81m ) in terms of v and w.
n
81m
Diberi log3 m = v dan log3 n = w, ungkapkan log9 ( ) dalam sebutan v dan w.
n
Solution:
Hint: Change the base first! Then separate the fraction!!

(c) Solve the equation / Selesaikan persamaan [3 marks] [Ans: 0.1813]


3(53x + 1) = 36
Answer space
Hint: Move the 3 to right side first! Then use log10

(d) 3 n  2  9 n 3
Simplify / Permudahkan [3 marks] [Ans: 1/3]
27 n3
Answer space
Hint: change all to base 3

(e) Solve 27t = 9(3t – 1), find the value of t / Selesaikan 27t = 9(3t – 1), cari nilai t . [3 marks] [Ans: t = ½]
Answer space

41
10 (a) Given that log8 n = 1 , find the value of n / Diberi log8 n = 1 , cari nilai n. [1 mark] [Ans: 2]
3 3
r s t
(b) Given that 2 = 3 = 6 , express t in terms of r and s. [4 marks] [Ans: t = rs/(s+r)]
Diberi 2r = 3s = 6t, ungkapkan t dalam sebutan r dan s.
(c) Given that y = kxm where k and m are constants, y = 4 when x = 2 and y = 8 when x = 5, find the values of k and m.
Diberi y = kxm dengan keadaan k dan m ialah pemalar , y = 4 apabila x = 2 dan y = 8 apabila x = 5, cari nilai k dan m.
[5 marks] [Ans: k = 2.368, m = 0.7565]
Answer space

11 The value of a house increases by 5% at the beginning of each year. If the initial value of the house is RM90 000, the
value of the house after t years, RM p, is given by p = 90000(1.05)t.
Nilai sebuah rumah meningkat sebanyak 5% pada setiap awal tahun daripada harga asal. Jika nilai rumah itu bermula dengan
RM90 000, selepas t tahun, didapati nilainya RM p, diberi oleh p = 90000(1.05)t.
Find / Cari
(a) the value of the house after 6.5 years / nilai rumah itu selepas 6.5 tahun, [1 mark] [Ans: 120608.61]
(b) minimum number of years for the value of the house to be more than RM150 000 [4 marks] [Ans: 11 years]
bilangan tahun minimum untuk nilai rumah itu melebihi RM150 000.
Answer space

12 Solve the simultaneous equations / Selesaikan persamaan serentak [7 marks] [Ans: x = 2/9, y = 4/9]
log9 y = 1 + log3 x
9x = 3y
Answer space

42
FORM 4 CHAPTER 6 COORDINATE GEOMETRY
PAPER 2 SECTION A / B
1 A straight line passes through the points A(– 2, – 5) and B(6, 7).
Suatu garis lurus melalui titik A(– 2, – 5) dan B(6, 7).
(a) Point D divides the line segment AB internally in the ratio 1 : 3. Find the coordinates of D.
Titik D membahagikan dalam tembereng garis AB dalam nisbah 1 : 3. Cari koordinat D.
Solution
Remember the formula for a point dividing a segment between two point with ratio:
nx  mx 2 ny1  my 2
The point which divide, (x, y) = ( 1 , )
mn mn
Do labeling first!!
Let A(– 2, – 5) = (x1, y1), B(6, 7) = (x2, y2), ratio 1 : 3 = m : n and the point D(x, y).
(3)( 6)  (1)( 6) (3)( 5)  (1)( 7)
So, point D = ( , )
1 3 1 3
= (6, – 2)
(b) Given that the point C(h, 10) lies on the straight line AB, find the value of h.
Diberi titik C(h, 10) terletak di atas garis lurus AB, cari nilai h.
Solution
y 2  y1
Remember the formula of gradient → m =
x 2  x1
Lie on the straight line → same gradient
Is parallel to….. → same gradient

C lies on the straight line AB → same gradient


So, gradient AC = gradient BC ←can also gradient AB = gradient AC = Gradient BC
10  (5) 10  7
=
h  (2) h6
15 3
=
h2 h6
Cross multiply: 15(h – 6) = 3(h + 2)
15h – 90 = 3h + 6
15h – 3h = 6 + 90
12h = 96
h = 96/12
h=8
(c) The straight line 8x + 4hy – 6 = 0 is perpendicular to the straight line 3x + y = 16. Find the value of h.
Garis lurus 8x + 4hy – 6 = 0 adalah berserenjang dengan garis lurus 3x + y = 16. Cari nilai h.
Solution
Line A which is perpendicular to Line B
↑ ↑
m1 m2
Perpendicular → m1 × m2 = –1

Gradient of straight line 8x + 4hy – 6 = 0 Gradient of straight line 3x + y = 16


4hy = – 8x + 6 y = – 3x + 16
8 x  6
y= m2 = – 3
4h
2 3
y= x
h 2h
2
m1 =
h

For perpendicular: m1 × m2 = –1
2
( ) × (– 3) = –1
h
6
=1
h
h=6

43
2 (a) The straight line x  y  1 has a y-intercept of 3 and is parallel to the straight line y + nx = 0. Determine the value
14 m
of m and of n.
Garis lurus x  y  1 mempunyai pintasan-y = 3 dan selari dengan garis lurus y + nx = 0. Tentukan nilai m dan n.
14 m
Solution
Remember the general equation of straight line → y = mx + c m = gradient c = y-intercept
And parallel → same gradient

Change x  y  1 to general form for straight line y = mx + c For straight line y + nx = 0.


14 m y = – nx
Multiply with 14m: mx + 14y = 14m Gradient is – n
14y = – mx + 14 m
y = mx  14m
14
y = m x + m
14
Given y-intercept is 3. So, the m = 3. The gradient is m = 3
14 14
Parallel means same gradient! So, 3 = – n
14
n= 3
14
(b) Given the points A(1, 8), point B(5, 4) and C(0, 1) are the vertices of a triangle.
Diberi titik-titik A(1, 8), titik B(5, 4) dan titik C(0, 1) merupakan bucu-bucu bagi suatu segitiga
(i) Given (3, b) is the midpoint of the points A and B. Find the value of b.
Diberi (3, b) ialah titik tengah bagi titik-titik A dan B. Cari nilai bagi b.
Solution

Remember the formula → Midpoint (x, y) = ( x1  x 2 , y1  y 2 )


2 2

Label the A(1, 8) = (x1, y1), point B(5, 4) = (x2, y2). Midpoint (x, y) = ( x1  x 2 , y1  y 2 )
2 2
(3, b) = ( 1  5 , 8  4 )
2 2
(3, b) = (3, 6)
So, b = 6
(ii) Find the distance between point A and point B / Cari jarak antara titik A dan titik B.
Solution
Remember the formula → Distance between two points = ( x2  x1 ) 2  ( y 2  y1 ) 2
Label the A(1, 8) = (x1, y1) and point B(5, 4) = (x2, y2).
Distance between AB = ( x2  x1 ) 2  ( y2  y1 ) 2
= (5  1) 2  (4  8) 2
= 32
(iii) Point Q moves such that the distance between the QB is always 4 units. Find the locus of Q.
Titik Q bergerak dengan keadaan jarak antara QB adalah sentiasa 4 unit. Cari lokus Q.
Solution
Remember the formula to find locus → Use the formula of distance ( x2  x1 ) 2  ( y 2  y1 ) 2

Let B(5, 4) = (x1, y1) and Q(x, y) = (x2, y2). So, ( x2  x1 ) 2  ( y 2  y1 ) 2 = 4


( x  5) 2  ( y  4) 2 = 4
Taking square for both sides: (x – 5)2 + (y – 4)2 = 16
(x – 5)(x – 5) + (y – 4)(y – 4) = 16
x2 – 10x + 25 + y2 – 8y + 16 = 16
x2 + y2 – 10x – 8y + 25 = 0
(iv) Find the area of triangles ABC / Cari luas segitiga ABC.
Solution
1 x1 x 2 x3 x1
Remember the formula of area of triangle → Area = │ │ ← closed point
2 y1 y 2 y3 y1
1 1 5 0 1
Area of triangles ABC = │ │
2 8 4 1 8
= ½ │(1×4 + 5×1+0×8) – (8×5+4×0+1×1)│
= ½ │ 9 – 41│
= 16 unit2
44
EXERCISE
1 Diagram 1 shows a kite ABCD.
Rajah 1 menunjukkan satu lelayang ABCD.
y
C(4, 15)

B(11, 4)
A(– 2, 3)
x
O
Diagram 1 / Rajah 1
Find / Cari:
(a) the coordinates of the midpoint of diagonal AC. [1 mark] [Ans: (1, 9)]
koordinat titik tengah pepenjuru AC.
(b) the equation of diagonal BD. [3 marks] [Ans: 2y = – x + 19]
persamaan pepenjuru BD .
Solution space

2 In Diagram 2, ACD and BCE are straight lines. C is the midpoint of AD and BC : BE = 1 : 5.
Dalam Rajah 2, ACD dan BCE ialah garis lurus. C ialah titik tengah AD dan BC : BE = 1 : 5.
y
D (5, 6)
B (2, 9/2)

A (1, 2) E
x
O
Diagram 2 / Rajah 2
Find / Cari
(a) the coordinates of point C. [2 marks] [Ans: (3, 4)]
koordinat titik C.
(b) the coordinates of point E. [2 marks] [Ans: (7, 2)]
koordinat titik E.
Solution space

45
3 Diagram 3 shows straight line CD which meets straight line AB at the point E. Point C lies on the y-axis.
Rajah 3 menunjukkan garis lurus CD bertemu garis lurus AB di titik E. Titik C terletak pada paksi-y.
y

x
O B (12, 0)
A (0, -3) E

D
Diagram 3 / Rajah 3
(a) Write down the equation of AB in the form of intercepts. [1 marks] [Ans: x/12 – y/3 = 1]
Tulis persamaan AB dalam bentuk pintasan.
(b) Given that 2AE = EB, find the coordinates of E. [1 marks] [Ans: (4, -2)]
Diberi 2AE = EB , cari koordinat E.
(c) Given that CD is perpendicular to AB, find the y-intercept of CD. [3 marks] [Ans: 14]
Diberi CD adalah berserenjang dengan AB, cari pintasan-y bagi CD.
Answer space

4 Diagram 4 shows a triangle AOB where O is the origin. Point C lies on the straight line AB.
Rajah 4 menunjukkan segitiga AOB di mana O adalah asalan. Titik C terletak pada garis lurus AB.
y
A (– 2, 5)

C
x
O
B (5, – 1)
Diagram 4 / Rajah 4
(a) Calculate the area, in unit2, of triangle AOB. [2 marks] [Ans: 23/2]
Hitung luas, dalam unit2, bagi segitiga AOB.
(b) Given that AC : CB = 3 : 2, find the coordinates of C. [2 marks] [Ans: (11/5, 7/5)]
Diberi AC : CB = 3 : 2, cari koordinat C.
(c) A point P moves such that its distance from point A is always twice its distance from point B.
Titik P bergerak dengan keadaan jaraknya dari titik A sentiasa dua kali jaraknya dari titik B.
(i) Find the equation of the locus of P. [3 marks] [Ans: 3x2 + 3y2 – 44x + 18y + 75]
Cari persamaan bagi lokus P.
(ii) Hence, determine whether the locus of P intersects the y-axis. [3 marks] [Ans: Tidak]
Seterusnya , tentukan sama ada lokus P memintas paksi–y.
Answer space

46
CHAPTER 9 DIFFERENTIATION
PAPER 2 SECTION A / B
1 (a) x2  4
/ Cari had x  4 .
2
Find the lim
x2 x  2 x  2 x2
Solution
Clue: If x is near to certain number → factorise and simplify first before substitution!
2 2 2
x 4
lim = lim x  2
x2 x  2 x2 x  2
( x  2)(x  2) ← Remember the formula a2 – b2 = (a + b)(a – b)
= lim
x2 x2
= lim ( x  2)
x2
= (–2 – 2)
=–4
(b) 2
1 2 x2  1
Find the lim ( 2 x 2
) / Cari had ( 2
).
x  x  3x x x  3x
Solution
Clue: If x is near to infinity → divide each term by x n before substitution!
2 x2  1
2x2  1 2 2
lim ( 2
)= lim ( xx 3xx2 )
x  x  3x x 2  2
x x
2  12
= lim ( x )
1 3
x  x
20
= ( )
03
2
=
3
(c) 2 2
Find the lim ( 3x  2 x ) / Cari had ( 3x  2 x ).
x 0x x x0
Solution
Clue: If x is near to zero → divide each term by x n before substitution!

3x  2 x 2 3x 2 x 2
lim ( ) = lim (  )
x 0 x x 0 x x
= lim (3  2 x)
x0
= 3+2(0)
=3
(d) Find the lim (3x  5) / Cari niai bagi lim (3x  5) .
x0 x0
Solution
Clue: If it is not in fraction form → direct substitution!

lim (3x  5) = 3(0) + 5


x0
=5
2 dy dy
Find the by using the first principle / Cari dengan menggunakan kaedah pertama.
dx dx
(a) y=5
(b) y = 3x2 – 2
(c) y=
4
x

47
3 Find the dy for / Cari dy untuk:
dx dx
(a) y = 3x2 – 8 (b) y = 6x1/2 – 4x3 + 5x – 1
dy Answer space
Apply the: y = xn, then = nxn – 1
dx
Answer space

4 Find the
dy
for / Cari
dy
untuk:
dx dx
(a) y=
2
– 2x5 + 8
– 2x (b) y= 3
+
2 32
x –2
x 3 2 3
x 5x
Move up the x first before differentiation!! Answer space
Answer space

5 Find the
dy
for / Cari
dy
untuk:
dx dx
(a) y = 3(2x – 8) + 4x2(x + 2) (b) y = (3x – 1)(x + 5)
Expand first before differentiation!! Answer space
Answer space

6 Find the
dy
for / Cari
dy
untuk:
dx dx
(a) y = (3x – 1)5 + 3x (b) y = 4(x2 + 3)3 + 3(x + 2)
Diff the power first! Then multiply with the diff inside!! Answer space
dy
Formula: y = (ax + b)n, = n(ax + b)n –1(a)
dx
Answer space

48
7 Find the dy for / Cari dy untuk:
dx dx
(a) y = (x + 5)(3x – 1)3 (b) y = (4x3 + 2x)(2x + 1)4
Answer space
Formula: If y = uv, then dy = u dv  v du
dx dx dx
Answer space

8 Find the
dy
for / Cari
dy
untuk:
dx dx
(a) y= 3x (b) 6 x2  5
x2  1
y=
(2 x  3)2
u dy v du  u dv
Formula: If y = , then = dx dx
v dx v2
Answer space

9 Given that f (x) = x(5 – 3x)2, find f '(2) / Diberi f (x) = x(5 – 3x)2, cari f '(2). [3 marks] [Ans: 13]
Answer space

10 The gradient of the tangent to the curve y = x2 (px  4) at x =  1 is 14. Find the value of p. [3 marks] [Ans: 2]
Kecerunan tangen kepada lengkung y = x2 (px  4) di x =  1 ialah 14. Cari nilai p.
Answer space
Remember: Gradient means dy/dx

49
11 The point P lies on the curve y = (x  5)2. It is given that the equation of the normal to the curve at P is y = – 1
4
x+ 23
4
.
Find the coordinates of P. [4 marks] [Ans: (7,4)]
Titik P terletak pada lengkung y = (x  5)2. Diberi bahawa persamaan normal kepada lengkung pada P ialah y = – 14 x + 23
4
. Cari
koordinat P.
Answer space
Remember: Normal means m1m2 = – 1 m1 is the dy/dx for first equation m2 is the dy/dx for second equation

12 The normal to the curve y = x2 + 1 at point P is parallel to the straight line 2y + x = 8. Find the equation of the normal to
the curve at point P. [4 marks] [Ans: x + 2y = 5]
Garis normal kepada lengkung y = x2 + 1 pada titik P adalah selari dengan garis lurus 2y + x = 8. Cari persamaan garis normal
kepada lengkung itu pada titik P.
Answer space
Remember: Normal means m1m2 = – 1 , Parallel means same gradient , equation means y = mx + c

13 A cube expands in such a way that its sides change at a rate of 2cms-1. Find the rate of change of the total surface area
when its volume is 125 cm3. [4 marks] [Ans: 120]
Sebuah kubus mengembang dengan keadaan sisinya berubah pada kadar 2 cms-1. Cari kadar perubahan jumlah luas permukaan
apabila isi padunya adalah 125 cm3.
Answer space
Apply chain rule……… dy
dx
= dy
d ....
× d....
dx

14 The curve y = –3x2 – 12x + 7 has a maximum point at x = k, such that k is a constant. Find the value of k.[3 marks][Ans:-2]
Lengkung y = –3x2 – 12x + 7 mempunyai titik maksimum pada titik x = k, dengan keadaan k adalah pemalar. Cari nilai k.
Answer space
Remember: Maximum or minimum point means dy/dx OR do the completing the square method!

50
15 The volume of water, V cm3 , in a container is given by V = 1 h3 + 7h, where h cm is the height of the water in the
3
container. Water is poured into the container at a rate of 8 cm3s-1. Find the rate of change of the height of water, in cms-1,
at the instant when its height is 3 cm. [3 marks] [Ans: ½ cms-1]
Isi padu air, V cm3 bagi sebuah bekas diberi oleh V = 1
3
h3 + 7h, dengan keadaan h cm adalah tinggi aras air dalam bekas. Air
dituang ke dalam bekas pada kadar 8 cm3 s-1. Cari kadar perubahan tinggi aras air dalam bekas, dalam cm s -1, pada ketika tingginya
adalah 3 cm.
Answer space

16 The tangent to the curve y = x3 + ax2  4x + 3 at x = 1 is parallel to the line y = 3x.


Tangen kepada lengkung y = x3 + ax2  4x + 3 pada x = 1 adalah selari dengan garis y = 3x.
Find/ / Cari
(a) the value of a / nilai a, [2 marks] [Ans: 2]
(b) the equation of tangent to the curve at x = 1 / persamaan tangen kepada lengkung pada x = 1. [3 marks] [Ans: y = 3x – 1]
(c) the x-coordinate at the turning points / koordinat-x bagi titik-titik pusingan. [3 marks] [Ans: 2/3, -2]
Answer space

17 Diagram shows a piece of wire bent to form the perimeter OABC of a sector of a circle, centre O, radius r cm, where
angle AOC is θ radian. The wire is of length 100 cm and r and θ may vary.
Rajah menunjukkan seutas dawai yang dibengkokkan menjadi perimeter sebuah sektor bulatan OABC berpusat O dan berjejari r cm
dengan keadaan sudut AOC adalah θ radian. Panjang dawai adalah 100 cm manakala r and θ akan berubah.
B

C
A
r cm
r cm

O
(a) Show that the area of the sector, A cm2 can be expressed as 50r  r2. [3 marks]
Tunjukkan bahawa luas sector, A cm2 boleh diungkapkan sebagai 50r  r2.
(b) Find / Cari
(i) the value of r for which the area enclosed by the wire is a maximum. [2 marks] [Ans: 25]
nilai r supaya luas yang dilingkungi oleh dawai adalah maksimum.
(ii) the corresponding value of θ in radian. [2 marks] [Ans: 2]
nilai sepadan bagi θ dalam radian .
Answer space

51
18 The curve y = x3 + 6x2  9x +1 passes through the point A(2, 1) and has two turning points, P(3, 1) and Q.
Lengkung y = x3 + 6x2  9x + 1 melalui titik A(2, 1) dan mempunyai dua titik pusingan, P(3, 1) dan Q.
Find / Cari
(a) the gradient of the curve at A / kecerunan lengkung pada titik A [3 marks] [Ans: 3]
(b) the equation of normal to the curve at A / persamaan normal kepada lengkung itu pada titik A [3 marks] [Ans:x+3y+1=0]
(c) the coordinates of Q and determine whether Q is a maximum or minimum point. [4 marks] [Ans: (1, -3), minimum]
koordinat Q dan tentukan sama ada Q adalah titik maksimum atau titik minimum.
Answer space

19 The diagram shows a water trough of semicircular cross-section, radius r m. Its length is l m. The total amount of
material used to make the trough is 20 m2.
Rajah menunjukkan satu bekas air dengan keratan rentas berbentuk semi bulatan, berjejari r m. Panjang bekas air adalah l m.
Jumlah bahan yang digunakan untuk membuat bekas air adalah 20 m2.
[Volume of cylinder / Isipadu silinder = r2h ]

lm

rm

(a) Show that the volume, V m3, of the trough is given by V = 10 r – 1


2
πr3. [3 marks]
3 3
Tunjukkan bahawa isipadu bekas air itu, V m , diberi oleh V = 10 r – 1
2
πr .
(b) Find the maximum volume of the trough. Give your answer correct to 2 decimal places in m3. [5 marks] [Ans: 9.71]
Show that this volume is indeed the maximum.
Cari isi padu maksimum bekas itu. Berikan jawapan anda tepat kepada dua tempat perpuluhan dalam m3.
Tunjukkan isipadu ini sebenarnya adalah maksimum.
Answer space

20 Find the value of 8.98 without using calculator.


Answer space

52
FORM 5 CHAPTER 4 VECTORS
PAPER 2 SECTION A / B
1

In Diagram 1, point Q lies on the straight line OR such that OQ : QR = 3 : 2. Point Q also is the midpoint of
the straight line BC. Given that = 6x, = 3y and = 4y.
Dalam Rajah 1, titik Q terletak pada garis lurus OR dengan keadaan OQ : QR = 3 : 2. Titik Q juga ialah titik
tengah bagi garis lurus BC. Diberi bahawa = 6x, = 3y dan = 4y.
(a) Express in terms of x and y:
Ungkapkan dalam sebutan x dan y:
(i) ,
(ii) ,
(iii) [6 marks / markah]
(b) Hence, determine whether is parallel to . [6 marks / markah]
Seterusnya, tentukan sama ada adalah selari dengan .
Answer Space

53
2 Diagram 2 shows a triangle OPR. The straight lines OQ and PS intersect at point T.
Rajah 2 menunjukkan segitiga OPR. Garis lurus OQ dan PS bersilang pada titik T.

Diagram 2 / Rajah 2
It is given that 5 =2 ,8 =3 , = 8x and = y.
Diberi bahawa 5 =2 ,8 =3 , = 8x dan = y.
(a) Express in terms of x and y; [5 marks / markah]
Ungkapkan dalam sebutan x dan y:
(i) ,
(ii) ,
(iii) .
(b) Given that =k and =h , express [2 marks / markah]
Diberi bahawa =k dan =h , ungkapkan
(i) in terms of k, x and y,
(ii) in terms of h, x and y,
(c) Hence, find the value of h and k. [3 marks / markah]
Answer Space

54
3 Diagram 3 shows a triangle PQR. The point S lies on PR and the point T lies on PQ.
The straight line QS intersects the straight line RT at the point U.
Rajah 3 menunjukkan segi tiga PQR. Titik S terletak pada PR dan titik T terletak pada PQ.
Garis lurus QS bersilang dengan garis lurus RT pada titik U.

Diagram 3 / Rajah 3
It is given that PRQ = 90º, = 9 x, = 12 y, : = 3 : 1 and : = 5 : 1.
Diberi bahawa PRQ = 90º, = 9 x, = 12 y, : = 3 : 1 dan : = 5 : 1.
(a) Express in terms of x and y; [3 marks / markah]
Ungkapkan dalam sebutan x dan y:
(i) ,
(ii) ,
(b) Using =h and =k , where h and k are constants, find the value of h and k. [5 marks]
Menggunakan =h dan =k , dengan keadaan h dan k ialah pemalar, cari nilai h dan k. [5 markah]
(c) Given that  x  = 5 units and  y  = 2 units, find  . [2 marks / markah]
Diberi bahawa  x  = 5 units dan  y  = 2 units, cari  .
Answer Space

55
4 (a) Given that p = 12 i + (k + 8) j and q = (k – 5) i – 3 j . If p and q are parallel,
(i) Find the values of k,
Cari nilai-nilai k,
(ii) If k is negative, find  p .
Jika k adalah negatif, cari  p . [4 marks / markah]
(b) Diagram 4 shows a triangle PRS. Point T lies on the straight line SQ and point U lies on the straight line
SR. It is given that ST = 3TQ, PQ = PR and SU : SR = 2 : 5. Given that = x and = y.
Rajah 4 menunukkan satu segitiga PRS. Titik T terletak pada garis lurus SQ dan titik U terletak pada garis lurus
SR. Diberi bahawa ST = 3TQ, PQ = PR dan SU : SR = 2 : 5. Diberi bahawa = x dan = y.

Diagram 4 / Rajah 4
Express in terms of x and y:
(i) ,
(ii) .
Answer Space

56
5 Diagram 5 shows triangle OPQ. The point S lies on OQ and the point R lies on OP.
The straight line PS intersects the straight line QR at the point T.
Rajah 5 menunjukkan segitiga OPQ. Titik S terletak pada OQ dan titik R terletak pada OP. Garis lurus PS bersilang
dengan garis lurus QR di titik T.

Diagram 5 / Rajah 5
It is given that OS = OQ, OR = OP, = x and = 3 y.
Diberi bahawa OS = OQ, OR = OP, = x dan = 3 y.
(a) Express in terms of x and y; [3 marks / markah]
Ungkapkan dalam sebutan x dan y:
(i) ,
(ii) . [3 marks / markah]
(b) Given that =m and = n , where m and n are constants, find the value of m and n. [7 marks]
Diberi =m dan =n , dengan keadaan m dan n ialah pemalar, cari nilai m dan n. [7 markah]
Answer Space

57
FORM 5 CHAPTER 5 TRIGONOMETRIC FUNCTIONS
PAPER 2 SECTION A / B
1 (a) Prove that 2 tan θ cos θ = sin 2θ.
2
[2 marks / markah]
Buktikan bahawa 2 tan θ cos2 θ = sin 2θ.
(b) Hence, solve the equation 4 tan θ cos2 θ = 1 for 0  θ  2π. [2 marks / markah]
Seterusnya selesaikan persamaan 4 tan θ cos2 θ = 1 for 0  θ  2π.
(c) (i) Sketch the graph of y = sin 2θ for 0  θ  2π.
Lakar graf y = sin 2θ untuk 0  θ  2π.
(ii) Hence, using the same axes, sketch a suitable straight line to find the number of solutions for the
equation 4π tan θ cos2 θ = x – 2π for 0  θ  2π. [6 marks / markah]
Seterusnya, dengan menggunakan paksi yang sama, lakar satu garis lurus yang sesuai untuk mencari
bilangan penyelesaian bagi persamaan 4π tan θ cos2 θ = x – 2π untuk 0  θ  2π.
Solution
(a) 2 tan θ cos2 θ = 2 ( ) cos2 θ [From tan θ = ]
= 2 sin θ cos θ [From sin 2θ = 2 sin θ cos θ  formula page]
= sin 2θ
(b) 4 tan θ cos2 θ = 1
2(2 tan θ cos2 θ) = 1  related to Question (a)
2 sin 2θ = 1
sin 2θ =
2θ = 30º (basic angle and in Quadrant I and II)
2θ = 30º or (180º – 30º)
2θ = 30º or 150º
θ = 15º or 75º
(c) (i) Plot y = 1 sin 2θ for 0  θ  2π

Amplitude shape 2 complete shape in 2π

(ii) 4π tan θ cos2 θ = x – 2π


2π (2 tan θ cos2 θ) = x – 2π  related to Question (a)
2π (sin 2θ) = x – 2π
2π y =  related to Question (c)(i)
y= –1
x 0 π  normally choose 0, π or 2π
y –1 –½
So, the number of solution is 4

Remember the general shape for 0  θ  2π


y = sin x y = cos x y = tan x

58
2 (a) Prove = tan 2x. [2 marks / markah]
Buktikan = tan 2x.
(b) Sketch the graph of y =  tan 2x  for 0  x  2π. [3 marks / markah]
Lakar graf bagi y =  tan 2x  untuk 0  x  2π.
(c) Hence, using the same axes, sketch a suitable straight line to find the number of solutions for the
equation   + = 1 for 0  x  2π.
State the number of solutions. [3 marks / markah]
Seterusnya, dengan menggunakan paksi yang sama, lakar satu garis lurus yang sesuai untuk mencari bilangan
penyelesaian bagi persamaan   + = 1 untuk 0  x  2π.
Nyatakan bilangan penyelesaian itu.
Solution
(a) =  sec2 x = 1 + tan2 x
=
=  cos 2x = 2cos2 x – 1
=
= tan 2x [##Always change sec //cosec //cot first, then change the terms with square]

(b) y =  tan 2x  for 0  x  2π



two general shapes in 2π

 
y = tan 2x y =  tan 2x 

(c)  + = 1 for 0  x  2π

 tan 2x  + =1  From (a): = tan 2x

y+ =1
y=1–

x 0 2π
y 1 0

So, there are 8 solutions.

59
3 (a) Prove that = sec2 x. [2 marks / markah]
2
Buktikan bahawa = sec x.
(b) (i) Sketch the graph of y = cos 2x + 1 for 0  x  2π. [3 marks / markah]
Lakar graf y = cos 2x + 1 untuk 0  x  2π.
(ii) Hence, using the same axes, sketch a suitable straight line to find the number of solutions for the
equation = + 1 for 0  x  2π.
State the number of solutions. [3 marks / markah]
Seterusnya, dengan menggunakan paksi yang sama, lakar satu garis lurus yang sesuai untuk mencari
bilangan penyelesaian bagi persamaan = + 1 untuk 0  x  2π.
Nyatakan bilangan penyelesaian itu.
Solution
(a) =  cos 2x = 2cos2 x – 1
=
=  sec x =
= sec2 x

(b) (i) y = cos 2x + 1



Two general shapes shift the x-axis 1 unit down

(ii) = + 1 for 0  x  2π
cos 2x + 1 = +1  From (a): = sec2 x
y= +1  From (b): y = cos 2x + 1

x 0 2π
y 1 1.5

So, there are 4 solutions.

60
4 (a) Sketch the graph of y = – 3 sin x for 0  x  2π. [3 marks / markah]
Lakar graf y = – 3 sin x untuk 0  x  2π.
(b) Hence, using the same axes, sketch a suitable graph to find the number of solutions for the equation
+ 3 sin x = 0 for 0  x  2π.
State the number of solutions. [3 marks / markah]
Seterusnya, dengan menggunakan paksi yang sama, lakar satu graf yang sesuai untuk mencari bilangan
penyelesaian bagi persamaan + 3 sin x = 0 untuk 0  x  2π.
Nyatakan bilangan penyelesaian itu.
Answer Space

5 (a) Sketch the graph of y = 1 + tan 2x for 0  x  π. [3 marks / markah]


Lakar graf y = 1 + tan 2x untuk 0  x  π.
(b) Hence, using the same axes, sketch a suitable graph to find the number of solutions for the equation
x + π tan 2x = 0 for 0  x  π.
State the number of solutions. [3 marks / markah]
Seterusnya, dengan menggunakan paksi yang sama, lakar satu graf yang sesuai untuk mencari bilangan
penyelesaian bagi persamaan x + π tan 2x = 0 untuk 0  x  π.
Nyatakan bilangan penyelesaian itu.
Answer Space

61
6 (a) Prove that sin (3x + ) – sin (3x – ) = cos 3x. [3 marks / markah]
Buktikan bahawa sin (3x + ) – sin (3x – ) = cos 3x.
(b) Hence,
Seterusnya,
(i) solve the equation sin (3x + ) – sin (3x – ) = for 0  x  π.
Give your answer in the simplest fraction form in terms of π rad. [4 marks / markah]
selesaikan persamaan sin (3x + ) – sin (3x – ) = untuk 0  x  π.
Beri jawapan anda dalam bentuk pecahan termudah dalam sebutan π rad.
(ii) sketch the graph of y = sin (3x + ) – sin (3x – ) – for 0  x  π. [3 marks / markah]
lakar graf bagi y = sin (3x + ) – sin (3x – ) – untuk 0  x  π.
Answer Space

7 (a) (i) Prove that 2 cos (x + 45º) cos (x – 45º) = cos 2x. [3 marks / markah]
Buktikan bahawa 2 cos (x + 45º) cos (x – 45º) = cos 2x.
(ii) Hence, solve the equation 2 cos (x + 45º) cos (x – 45º) = for 0  x  2π. [4 marks / markah]
Seterusnya, selesaikan persamaan 2 cos (x + 45º) cos (x – 45º) = untuk 0  x  2π.
(b) Sketch the graph of y = cos 2x for 0  x  2π. [3 marks / markah]
Lakar graf bagi y = cos 2x untuk 0  x  2π.
Answer Space

62
8 (a) Sketch the graph for y = 3 tan x for 0  x  2π. [3 marks / markah]
Lakar graf bagi y = 3 tan x untuk 0  x  2π.
(b) Hence, on the same axes, sketch a suitable straight line to determine the number of solution for the
equation 4 tan x = 1 for 0  x  2π. State the number of solutions. [3 marks / markah]
Seterusnya, pada paksi yang sama, lakarkan satu garis lurus yang sesuai untuk menentukan bilangan
penyelesaian bagi persamaan 4 tan x = 1 untuk 0  x  2π. Nyatakan bilangan penyelesaian itu.
Answer Space

9 (a) Prove that ( – )2 = 4(1 – sin 2x). [3 marks / markah]


Buktikan bahawa ( – 2
) = 4(1 – sin 2x).
(b) Hence, solve the equation ( – )2 = sin 2x for 0  x  π. [4 marks / markah]
Seterusnya, selesaikan persamaan ( – ) = sin 2x untuk 0  x  π.
2

Answer Space

63
10 (a) Given that sin x = , where x is an acute angle, determine the value of sin 2x without using the
mathematical tables or a calculator. [3 marks / markah]
Diberi bahawa sin x = , dengan keadaan x ialah sudut tirus, tentukan nilai bagi sin 2x tanpa menggunakan
jadual matematik atau kalkulator.
(b) Hence, or otherwise, show that sin 3x = . [3 marks / markah]
Answer Space

11 Diagram shows a graph of trigonometry function y = f (x) for 0  x  π.


Rajah menunjukkan satu graf bagi fungsi trigonometri y = f (x) untuk 0  x  π.

(a) Write an equation for the graph of trigonometric function y = f (x). [3 marks / markah]
Tulis satu persamaan untuk graf bagi fungsi trigonometri y = f (x).
(b) Sketch the graph of y =  f (x) + 1  for 0  x  π. [2 marks / markah]
Lakarkan graf y =  f (x) + 1  untuk 0  x  π.
(c) Hence, on the same axes, sketch a suitable straight line to determine the number of solution for the
equation [ f (x) + 1 – 2] = 1 for 0  x  π. State the number of solutions. [3 marks / markah]
Seterusnya, pada paksi yang sama, lakarkan satu garis lurus yang sesuai untuk menentukan bilangan
penyelesaian bagi persamaan [ f (x) + 1 – 2] = 1 untuk 0  x  π. Nyatakan bilangan penyelesaian itu.
Answer Space

64
12 (a) Sketch the graph for y =  sin x +  for 0  x  2π. [3 marks / markah]
Lakar graf bagi y =  sin x +  untuk 0  x  2π.
(b) Hence, on the same axes, sketch a suitable straight line to determine the number of solution for the
equation  sin x +  = 1 – for 0  x  2π. State the number of solutions. [3 marks / markah]
Seterusnya, pada paksi yang sama, lakarkan satu garis lurus yang sesuai untuk menentukan bilangan
penyelesaian bagi persamaan  sin x +  = 1 – untuk 0  x  2π. Nyatakan bilangan penyelesaian itu.
Answer Space

13 (a) Prove that = sin 2x. [2 marks / markah]


Buktikan = sin 2x.
(b) Sketch the graph of y = sin 2x for 0  x  π. [3 marks / markah]
Lakar graf bagi y = sin 2x untuk 0  x  π.
(c) Hence, using the same axes, sketch a suitable straight line to find the number of solutions for the
equation x– = for 0  x  π.
State the number of solutions. [3 marks / markah]
Seterusnya, dengan menggunakan paksi yang sama, lakar satu garis lurus yang sesuai untuk mencari bilangan
penyelesaian bagi persamaan x– = untuk 0  x  2π.
Nyatakan bilangan penyelesaian itu.
Answer Space

65
14 (a) Prove that = tan x. [2 marks / markah]
Buktikan = tan x.
(b) Sketch the graph of y = for 0  x  2π. [3 marks / markah]
Lakar graf bagi y = untuk 0  x  2π.
(c) Hence, using the same axes, sketch a suitable straight line to find the number of solutions for the
equation – = 1 for 0  x  2π.
State the number of solutions. [3 marks / markah]
Seterusnya, dengan menggunakan paksi yang sama, lakar satu garis lurus yang sesuai untuk mencari bilangan
penyelesaian bagi persamaan – = 1 untuk 0  x  2π.
Nyatakan bilangan penyelesaian itu.
Answer Space

15 (a) Sketch the graph of y =  3 sin 2x  for 0  x  2π. [3 marks / markah]


Lakar graf bagi y =  3 sin 2x  untuk 0  x  2π.
(b) Hence, using the same axes, sketch a suitable straight line to find the number of solutions for the
equation +  sin 2x  = 1 for 0  x  2π.
State the number of solutions. [3 marks / markah]
Seterusnya, dengan menggunakan paksi yang sama, lakar satu garis lurus yang sesuai untuk mencari bilangan
penyelesaian bagi persamaan +  sin 2x  = 1 untuk 0  x  2π.
Nyatakan bilangan penyelesaian itu.
Answer Space

66
CHAPTER 8 CIRCULAR MEASURE
PAPER 2 SECTION A / B
Recall the formula
Angle to radian Radian to angle Perimeter of circle, s Area of circle, A
 180 o s=2πr A=πr2
Rad = × θº Angle = × rad
180 o 

Length of arc, s = r θ Area of sector, A = ½ r 2 θ Area of shaded region, A Pythagoras thereom


A = ½ r 2 (θ – sinθº) c
r r A b
O s O r
a
O
B
c2 = a2 + b2
θ must in radian θ must in radian
θ must in radian tan θ = a/c cos θ = a/c
θº must in angle sin θ = b/a
1 Diagram 1 shows a circle PQRT, centre O and radius 10 cm. AQB is a tangent to the circle at Q. The straight lines, AO
and BO, intersect the circle at P and R respectively. OPQR is a rhombus. ACB is an arc of a circle, centre O.
Rajah 1 menunjukkan sebuah bulatan PQRT, berpusat di O dan berjejari 10 cm. AQB ialah tangen kepada bulatan itu di Q. Garis
lurus, AO dan BO, masing-masing bersilang pada P dan R. OPQR ialah sebuah rombus. ACB ialah lengkok bulatan itu berpusat di O.
C

A B
Q
P R
rad
cm
10
O

T
Diagram 1 / Rajah 1
Calculate / Hitung
(a) the angle , in terms of , / sudut  , dalam sebutan ,
Solution
PQRO is a rhombus meaning that the length of PO = OR = QR = QP which is 10 cm!
So for triangle PQO, Q
cm
The value of  POQ = 60º 10
= /3 P
10
Hence, the angle of α = 2 ×  POQ cm
O
= 2 ×/3
α = 2/3

(b) the length, in cm, of the arc ACB, / panjang, dalam cm, lengkok ACB
Solution
Q B
From cos 60º = QO
OB
10
cos 60º = OB 10 cm o
60
OB = 10
cos 60o
O
OB = 20 cm
α = 2/3
= 120º
So, length of arc ACB = r α α in radian
= 20(2.0952) =  o × 120º
180
= 41.9048 cm = 2.0952 rad.
(c) the area, in cm2, of the shaded region / luas, dalam cm2 kawasan berlorek
Solution
Area of the shaded region = Area of sector OACB – Area of triangle OAB
= ½ (20)2(2.0952) – ½ × AO × OB × sin α
= ½ (20)2(2.0952) – ½ × 20 × 20 × sin 120º
= 245.8349 cm2

67
2 In Diagram 2, PBQ is a semicircle with centre O and has a radius of 10 m. RAQ is a sector of a circle with centre A and
has a radius of 16 m.
Dalam Rajah 2, PBQ ialah separuh bulatan berpusat di O dan mempunyai jejari 10 m. RAQ ialah sektor bulatan berpusat di A dan
mempunyai jejari 16 m.
R

P
A O Q
Diagram 2 / Rajah 2
It is given that AB = 10 m and  BOQ = 1.876 radians. [Use  = 3.142]
Diberi bahawa AB = 10 m dan  BOQ = 1.876 radian. [Guna  = 3.142]
Calculate / Hitung
(a) the area , in m2 , of sector BOQ,
luas, dalam m2, sector BOQ,
Solution
Area of sector BOQ = ½ r 2 θ B
= ½ × 102 × 1.876
= 93.8 m2

1.876 rad
Q
O 10 m
(b) the perimeter, in m, of the shaded region,
perimeter, dalam m, kawasan berlorek,
Solution
Perimeter BQ = BO ×  BOQ in radian [s = r θ]
= 10 × 1.876
= 18.76 m

Length of BR = AR – AB
= 16 m – 10 m
=6m

Length of arc RQ = AR ×  BAQ in radian  BAQ =  AOB = 180º –  BOQ


= 16 × 1.266 = 72.527º
= 20.256 m = 1.266 rad

So, perimeter of the shaded region = Perimeter BQ + Length BR + Length of arc RQ


= 18.76 m + 6 m + 20.256 m
= 45.016 m

(c) the area , in m2 , of the shaded region .


luas, dalam m2, kawasan berlorek,
Solution
Area of sector RAQ = ½ × AR2 ×  RAQ
= ½ × 162 × 1.266
= 162.048 m2

Area of sector BOQ = 93.8 m2 [From Question 2(a)]

Area of triangle ABO = ½ × BA × BO × sin  ABO in angle


= ½ × 10 × 10 × sin34.946º
= 28.6402 m2

So, area of the shaded region = Area of sector RAQ – Area of sector BOQ – Area of triangle ABO
= 162.048 m2 – 93.8 m2 – 28.6402 m2
= 39.6078 m2

68
EXERCISE
1 Diagram 1 shows a circle, centre O and radius 20 cm inscribed in a sector PAQ of a circle, centre A. The straight lines,
AP and AQ, are tangents to the circle at point B and point C, respectively. [Use π = 3.142]
Rajah 5 menunjukkan sebuah bulatan, berpusat di O dan berjejari 20 cm. Garis lurus, AP dan AQ masing-masing ialah tangen
kepada bulatan itu di titik B dan titik C. [Guna π = 3.142]

P O Q
20 cm
B C

o
60

A
Diagram 1 / Rajah 1
Calculate / Hitung
(a) the length, in cm, of the arc PQ, [5 marks] [Ans: 62.84]
panjang , dalam cm, lengkok PQ,

(b) the area, in cm2, of the shaded region. [5 marks] [Ans: 354.513]
luas, dalam cm2, kawasan berlorek.
Answer space

69
2 Diagram 2 shows a semicircle OABCD with centre O and a right angled triangle ADE.
Rajah 2 menunjukkan separuh bulatan OABCD berpusat di O dan segitiga bersudut tepat ADE.
E C
B

10 cm

A O 6 cm D
Diagram 2 / Rajah 2

It is given that the length of AE = 10 cm and the radius of the semicircle OABCD is 6 cm.
Diberi bahawa panjang AE = 10 cm dan jejari separuh bulatan OABCD ialah 6 cm. [Use / Guna π = 3.142]
Calculate / Hitung
(a)  EDA in radian [2 marks] [Ans: 0.6948]
 EDA dalam radian,
(b) (i) the length , in cm, of the arc AB, [2 marks] [Ans: 8.338]
panjang, dalam cm, lengkok AB,
(ii) the perimeter, in cm , of the shaded region, [3 marks] [Ans: 24.74]
perimeter, dalam cm, kawasan berlorek,
(c) the area, in cm2, of the segment BCD. [3 marks] [Ans: 13.84]
luas, dalam cm2 , tembereng BCD.
Answer space

70
INTENSIVE EXERCISE
PAPER 2 SECTION B
1 Two circles of equal sides, each with a radius of 5 cm, touch each internally. Both of them touch another
circle, centre O and radius 15 cm internally as shown in the Diagram 1.
Dua bulatan dengan sisi sama, setiap dengan jejari 5 cm, bersentuh secara dalam. Kedua-duanya bersentuh dengan satu
lagi bulatan, pusat O dan jejari 15 cm secara dalaman seperti ditunjukkan dalam Rajah 1.

Diagram 1 / Rajah 1
Calculate the area of the shaded region. [5 marks / markah]
Hitung luas kawasan berlorek.
Answer Space

71
2 Diagram 2 shows circular section of a uniform log with a radius of 50 cm floating in water. The point A and B
are on the surface and the highest point C is 10 cm above the water surface. Calculate
Rajah 2 menunjukkan keratan bulat bagi suatu kayu balak seragam dengan jejari 50 cm yang terapung di air. Titik A
dan B adalah di permukaan air dan titik tertinggi C ialah 10 cm di atas permukaan air. Hitung

Diagram 2 / Rajah 2
(a) AOB in radians, [3 marks / markah]
AOB dalam radian,
(b) the length of the arc ACB, [2 marks / markah]
Panjang lengkok ACB,
(c) the area of the cross-section below the water surface. [3 marks / markah]
luas keratan rentas di bawah permukaan air.
Answer Space

72
3 In Diagram 3, PXQ is an arc of a circle at centre O and radium 15 cm with POQ = 1.3 radians. PYQ is an arc
of a circle with centre R and radius 10 cm. Calculate
Dalam Rajah 3, PXQ ialah suatu lengkok bulatan pada pusat O dan jejari 15 cm dengan POQ = 1.3 radian. PYQ ialah
suatu lengkok bulatan dengan pusat R dan jejari 10 cm. Hitung

Diagram 3 / Rajah 3
(a) the length of the chord PQ,
panjang perentas PQ,
(b) the value of θ in radians,
nilai θ dalam radians,
(c) the difference in length between arcs PYQ and PXQ.
beza panjang antara lengkok PYQ dan PXQ.
Answer Space

73
4 Diagram 4 shows the two arcs AB and CD, of concentric circles, centre O and radii 2k cm and 3k cm
respectively. Given that AOB = rad and the perimeter of ABCD is 18 cm, find
Rajah 4 menunjukkan dua lengkok AB dan CD, dengan pusat sama O dan jejari 2k cm dan 3k cm masing-masing. Diberi
bahawa AOB = rad dan perimeter ABCD ialah 18 cm, cari

Diagram 4 / Rajah 4
(a) the value of k,
nilai k,
(b) the difference between the lengths of the arcs AB and CD.
beza panjang antara lengkok AB dan CD.
Answer Space

74
5

Diagram 5 shows a semicircular wall light OHJK with centre O. JG is tangent to the circle at J.
Rajah 5 menunjukkan sebuah lampu dinding berbentuk semi bulatan OHJK yang berpusat O. JG ialah tangent kepada
bulatan di J.
Given the diameter HK = 16 cm dan  HOJ = radians, calculate the area of the shaded region.
Diberi diameter HK = 16 cm dan  HOJ = radians, hitung luas rantau berlorek.
[Use / Guna = 3.142] [4 marks / markah]
Answer Space

Diagram 6 / Rajah 6
Diagram 6 shows a long jump field OABCDE. ABDE is a rectangle with AE =12 cm and AB = 8 cm. OJK and
OBCD are sectors with common centre O. Students who can jump into the shaded region from O will be
chosen to represent the school. It is given that OK : KB = 4 : 1. Using  = 3.142, calculate the perimeter, in
cm, of the shaded region.
Rajah 6 menunjukkan satu padang lompat jauh OABCDE. ABDE ialah segiempat tepat dengan AE = 12 cm dan AB =
8cm. OJK dan OBCD ialah sektor-sektor bulatan dengan pusat sepunya O. Murid-murid yang dapat melompat ke
kawasan berlorek dari O akan dipilih untuk mewakili sekolah. Diberi bahawa OK : KB = 4 : 1. Menggunakan π = 3.142,
hitung perimeter, dalam cm, kawasan berlorek. [4 marks / markah]
Answer Space

75
7

Diagram 7 / Rajah 7
Diagram 7 shows a circular lake with centre O. Given that diameter GH = 24 m and the length of arc HK= 15
m. Point J is right above H. Encik Kamal owns a villa in the shaded region HJK. The segment of GK is used
as fish rearing farm.
Rajah 7 menunjukkan sebuah tasik berbentuk bulatan yang berpusat di O. Diberi bahawa diameter GH = 24 m dan
panjang lengkok HK = 15 m. Titik J berada tegak di atas H. Encik Kamal memiliki sebuah villa di kawasan berlorek
HJK. Segmen GK digunakan sebagai tempat perternakan ikan.
Use/ Guna = 3.142,
(a) show that  = 0.625 radian. [3 marks / markah]
tunjukkan bahawa  = 0.625 radian.
(b) calculate the perimeter of fish farm GK. [3 marks / markah]
cari perimeter tempat ternakan ikan GK.
(c) calculate the area, in m2, of Encik Kamal's villa. [4 marks / markah]
hitung luas, dalam m2, villa Encil Kamal.
Answer Space

76
8

Diagram 8 / Rajah 8
Diagram 8 shows PQ and PR are tangent to the circle with centre O. OQR is a sector with centre O and NQR
is another sector with centre N.
Rajah 8 menunjukkan bahawa PQ dan PR ialah tangen kepada bulatan berpusat O. OQR ialah sektor bulatan berpusat
O dan NQR ialah sektor bulatan lain yang berpusat N.
It is given that OQ = 8 cm, QN= RN = 12 cm and OP = 17 cm.
Diberi bahawa OQ = 8 cm, QN = RN = 12 cm dan OP = 17 cm. (Use/ Guna π = 3.142)
(a) Show that  = 2.162 radians. [ 2 marks / markah]
Tunjukkan bahawa  = 2.162 radians.
(b) Find the perimeter, in cm, of segment NQ. [ 4 marks / markah]
Cari perimeter, dalam cm, segmen NQ.
(c) Calculate the area, in cm2, of the shaded region. [4 marks / markah]
Hitung luas, dalam cm2, kawasan berlorek.
Answer Space

77
9 Diagram 9 shows a cylindrical container with the length of 20 cm placed on the floor against the wall. Q is a
point on the edge of the base of the container. It is given that the distance of point Q is 2 cm from the wall and
1 cm from the floor.
Rajah 9 menunjukkan sebuah bekas berbentuk silinder dengan panjang 20 cm yang diletak di atas lantai dan rapat pada
dinding. Q ialah satu titik pada tepi tapak bekas itu. Diberi bahawa jarak titik Q adalah 2 cm dari dinding dan 1 cm dari
lantai.

Diagram 9 / Rajah 9
Mira wants to keep the container in a box with a dimension of 21 cm ×7 cm × 7 cm. Determine whether the
container can be kept in that box or otherwise. Give a reason for your answer.
Mira ingin menyimpan bekas itu ke dalam sebuah kotak yang berukuran 21 cm ×7 cm × 7 cm. Tentukan samada bekas
itu boleh disimpan ke dalam kotak itu atau sebaliknya. Beri sebab kepada jawapan anda. [6 marks / markah]
Answer Space

78
FORM5 CHAPTER 2: LINEAR LAW
PAPER 2 SECTION B
1) Equation of straight line: y = mx + c
2) Gradient, m = or m = –
3) c = y-intercept // Or choose one point = (x, y) and substitute into y = mx + c to find the c.
4) To find the non-linear equation
1st: Find the gradient, m
2nd: Find the c = y-intercept or choosing one point to be substituted into the y = mx + c
3rd: Use Y = mX + c, then substitute all with Y to y-axis, X to x-axis, m and c with values.

1 SPM 2017 Paper 2 Q9


Use a graph paper to answer this question.
Guna kertas graf untuk menjawab soalan ini.

Table 1 shows the values of two variables, x and y obtained from an experiment.
The variables x and y are related by the equation y – = , where h and k are constants.
Jadual 1 menunjukkan nilai-nilai bagi dua pembolehubah, x dan y yang diperolehi daripada suatu eksperimen .
Pembolehubah-pembolehubah x dan y dihubungkan oleh persamaan y – = , dengan keadaan h dan k ialah
pemalar.
x 1.5 2.0 3.5 4.5 5.0 6.0
y 4.5 5.25 5.5 6.3 6.34 6.5
Table 1 / Jadual 1
(a) Plot xy against x, using a scale of 2 cm to 1 unit on the x-axis and 2 cm to 5 units on the xy-axis.
Hence, draw the line of best fit. [4 marks / markah]
Plot xy melawan x, menggunakan skala 2 cm kepada 1 unit pada paksi-x dan 2 cm kepada 5 unit pada paksi-xy.
Seterusnya, lukis garis penyuaian terbaik.
(b) Using the graph in 1(a), find
Menggunakan graf di 1(a), cari
(i) the value of h and k,
nilai bagi h dan k,
(ii) the correct value of y if one of the values of y has been wrongly recorded during the experiment.
nilai yang betul bagi y jika salah satu nilai y telah direkod salah semasa eksperimen.
[6 marks / markah]
Answer Space
(a) Plot xy against x
 
y-axis x-axis
x 1.5 2.0 3.5 4.5 5.0 6.0
xy 6.75 10.50 19.25 28.35 31.70 39.00  All must be in 2 decimal places
On graph paper.

(b) From y – =
xy – x
= hk
xy = x + hk
xy = x + hk
Compare with Y = mX + c
= m and hk = c

(b)(i) =m and hk = c
= 7.2 (51.84)k = – 4
h = 51.84 k = 0.07716

(b)(ii) Wrong value for y happens when x = 3.5


From graph, xy = 21
3.5y = 21
y=6

79
GraphGraph
of sine i against
of xy against xsine r
Graf Graf
sin ixymelawan
melawan x sin r

xy

40
(6, 39)

35

30

25

Correct value
20

15
m=

m = 7.2
10

(1, 3)

0
1 2 3 4 5 6 x

–5
y-intercept, c = – 4

80
2 SPM 2018 Paper 2 Q11
Use a graph paper to answer this question.
Guna kertas graf untuk menjawab soalan ini.
Table 2 shows the values of two variables, x and y obtained from an experiment.
A straight line will be obtained when a graph of against is plotted.
Jadual 2 menunjukkan nilai-nilai bagi dua pembolehubah, x dan y yang diperolehi daripada suatu eksperimen .
Satu garis lurus akan diperolehi apabila satu graf melawan diplotkan.
x 1.25 1.43 2.00 2.50 4.00 5.00
y 4.47 4.38 4.18 3.87 2.83 2.24
Table 2 / Jadual 2
(a) Based on Table 2, construct a table for the values of and . [2 marks / markah]
Berdasarkan pada Jadual 2, bina satu jadual untuk nilai-nilai and .
(b) Plot against , using a scale of 2 cm to 1 unit on the -axis and 2 cm to 2 units on the -axis.
Hence, draw the line of best fit.
Plot melawan , dengan menggunakan skala 2 cm kepada 1 unit pada paksi- and 2 cm to 2 units on the
paksi- .
(c) Using the graph in 2(b)
Menggunakan graf di 2(b)
(i) find the value of y when x = 2.7
nilai y apabila x = 2.7
(ii) express y in terms of x. [5 marks / markah]
ungkapkan y dalam sebutan x.
Answer Space

81
3 SPM 2016 Paper 2 Q10
Use a graph paper to answer this question.
Guna kertas graf untuk menjawab soalan ini.
Table 3 shows the values of two variables, x and y obtained from an experiment.
Variables x and y are related by the equation y = 2px + , where p and q are constants.
Jadual 3 menunjukkan nilai-nilai bagi dua pembolehubah, x dan y yang diperolehi daripada suatu eksperimen .
Pembolehubah-pembolehubah x dan y dihubungkan oleh persamaan y = 2px + , dengan keadaan p dan q ialah
pemalar.
x 1 2 3 4 5 6
y 3.10 2.30 2.33 2.35 2.72 3.00
Table 3 / Jadual 3
(a) Based on Table 3, construct a table for the values of x2 and xy. [2 marks / markah]
Berdasarkan pada Jadual 3, bina satu jadual untuk nilai-nilai x2 dan xy.
(b) Plot xy against x2, using a scale of 2 cm to 5 units on the x2-axis and 2 cm to 2 units on the xy-axis.
Hence, draw the line of best fit. [3 marks / markah]
Plot xy melawan x2, menggunakan skala 2 cm kepada 5 units pada paksi-x2 dan 2 cm kepada 2 unit pada paksi-
xy. Seterusnya, lukis garis penyuaian terbaik.
(c) Using the graph in 3(b), find the value of
Menggunakan graf di 3(b), cari nilai
(i) p,
(ii) q. [5 marks]
Answer Space

82
4 SPM 2015 Paper 2 Q11
Use a graph paper to answer this question.
Guna kertas graf untuk menjawab soalan ini.
Table 4 shows the values of two variables, x and y obtained from an experiment.
Variables x and y are related by the equation nx = py + xy, where n and p are constants.
Jadual 4 menunjukkan nilai-nilai bagi dua pembolehubah, x dan y yang diperolehi daripada suatu eksperimen .
Pembolehubah-pembolehubah x dan y dihubungkan oleh persamaan nx = py + xy, dengan keadaan n dan p ialah
pemalar.
x 1.5 2.0 3.0 4.0 5.0 6.0
y 2.020 0.770 0.465 0.385 0.351 0.339
Table 4 / Jadual 4
(a) Based on Table 4, construct a table for the values of and . [2 marks / markah]
Berdasarkan Jadual 4, bina satu jadual untuk nilai-nilai dan .
(b) Plot against , using a scale of 2 cm to 0.1 unit on -axis and 2 cm to 0.5 unit on -axis.
Hence, draw the line of best fit. [3 marks / markah]
Plot melawan , menggunakan skala 2 cm kepada 0.1 unit pada paksi- dan 2 cm kepada 0.5 unit pada paksi-
. Seterusnya, lukis garis penyuaian terbaik.
(c) Using the graph in 4(b), find the value of
Menggunakan graf di 4(b), cari nilai bagi
(i) n,
(ii) p. [5 marks / markah]
Answer Space

83
5 SPM 2014 Paper 2 Q11
Use a graph paper to answer this question.
Guna kertas graf untuk menjawab soalan ini.
Table 5 shows the values of two variables, x and y obtained from an experiment. Variables x and y are
related by the equation y = , where h and k are constants.
Jadual 5 menunjukkan nilai-nilai bagi dua pembolehubah, x dan y yang diperolehi daripada suatu eksperimen .
Pembolehubah-pembolehubah x dan y dihubungkan oleh persamaan y = , dengan keadaan h dan k ialah pemalar.
x 4 6 8 10 12 14
y 2.82 2.05 1.58 1.23 0.89 0.66
Table 5 / Jadual 5
(a) Based on Table 5, construct a table for the values of log10 y. [2 marks / markah]
Berdasarkan pada Jadual 5, bina satu jadual untuk nilai-nilai log10 y.
(b) Plot log10 y against x, using a scale of 2 cm to 2 units on x-axis and 2 cm to 0.1 unit on log10 y-axis.
Hence, draw the line of best fit. [3 marks / markah]
Plot log10 y melawan x, menggunakan skala 2 cm kepada 2 unit pada paksi-x dan 2 cm kepada 0.1 unit pada
paksi-log10 y.
(c) Using the graph in 5(b), find the value of
Menggunakan graf di 5(b), cari nilai bagi
(i) y when x = 2,
y apabila x = 2,
(ii) h,
(iii) k. [6 marks / markah]
Answer Space

84
6 SPM 2013 Paper 2 Q7
Use a graph paper to answer this question.
Guna kertas graf untuk menjawab soalan ini.
Table 7 shows the values of two variables, x and y obtained from an experiment. Variables x and y are
related by the equation y = + 2kx, where p and k are constants.
Jadual 7 menunjukkan nilai-nilai bagi dua pembolehubah, x dan y yang diperolehi daripada suatu eksperimen .
Pembolehubah-pembolehubah x dan y dihubungkan oleh persamaan y = + 2kx, dengan keadaan p dan k ialah
pemalar.
x 1.0 2.0 3.0 4.0 5.0 5.5
y 8.00 4.75 3.83 3.58 3.65 3.72
Table 7 / Jadual 7
(a) Based on Table 7, construct a table for the values of x2 and xy. [2 marks / markah]
Berdasarkan pada Jadual 7, bina satu jadual bagi nilai-nilai x2 dan xy.
(b) Plot xy against x2, using a scale of 2 cm to 5 units on x2-axis and 2 cm to 2 units on the xy-axis.
Hence, draw the line of best fit. [3 marks]
Plot xy melawan x2, menggunakan skala 2 cm kepada 5 unit pada paksi-x2 dan 2 cm kepada 2 unit pada paksi-
xy. Seterusnya, lukis garis penyuaian terbaik.
(c) Using the graph in 6(b), find the value of
Menggunakan graf di 6(b), cari nilai bagi
(i) p,
(ii) k. [5 marks / markah]
Answer Space

85
7 TRIAL STATE JUJ PAHANG 2018
Use a graph paper to answer this question.
Guna kertas graf untuk menjawab soalan ini.

Diagram 7 / Rajah 7
Diagram 7 shows that AB is a smooth plane that slopes with θ to the horizontal. A particle P is rolled up
along the plane from point A with an initial velocity of v0 ms-1 and its distance from point A is s meter after t
seconds. The values for t and s are shown in Table 7.
Rajah 7 menunjukkan bahawa AB ialah satu satah licin dengan sudut satah θ kepada ufukan. Suatu zarah P diguling
naik sepanjang satah dari titik A dengan halaju awal v 0 ms-1 dan jaraknya dari titik A ialah s meter selepas t saat.
Nilai-nilai bagi t dan s ditunjukkan dalam Jadual 7.
t (second) 1 2 3 4 5 6
s (meter) 35.75 64 90 108 125 126
Table 7 / Jadual 7
The relation of the distance travelled, s meter with the time, t second is s = vot – 5t2 sin θ.
Hubungan antara jarak yang dilalui, s meter dengan masa, t saat ialah s = vot – 5t2 sin θ.
(a) Plot against t, using a scale of 2 cm to 1 unit on the t-axis and 2 cm to 5 units on the -axis.
Hence, draw the line of best fit. [4 marks / markah]
Plot melawan t, menggunakan skala 2 cm kepada 1 unit pada paksi-t dan 2 cm kepada 5 unit pada paksi- .
Seterusnya, luksi garis penyuaian terbaik.
(b) Using the graph in 7(b), find the value of
Menggunakan graf di 7(b), cari nilai bagi
(i) θ,
(ii) vo,
(iii) the distance of particle after 1.5 seconds from point A. [6 marks / markah]
jarak zarah selepas 1.5 saat dari titik A.
Answer Space

86
8 TRIAL STATE KELANTAN 2018
Use a graph paper to answer this question.
Guna kertas graf untuk menjawab soalan ini.
Table 8 shows the values of two variables, x and y obtained from an experiment. The variables x and y are
related by an equation hy = 2(hx)2 + kx, where h and k are constants.
Jadual 8 menunjukkan nilai-nilai bagi dua pembolehubah, x dan y yang diperolehi daripada suatu eksperimen.
Pembolehubah-pembolehubah x dan y dihubungkan oleh persamaan hy = 2(hx)2 + kx, dengan keadaan h dan k ialah
pemalar.
x 2 3 4 5 6 7
y 8 13.2 20 27.5 36.5 45.4
Table 8 / Jadual 8
(a) Based on Table 8, construct a table for the values of . [2 marks / markah]
Berdasarkan pada Jadual 8, bina satu jadual untuk nilai-nilai .
(b) Plot against x, using a scale of 2 cm to 1 unit on x-axis and 2 cm to 1 unit on the -axis.
Hence, draw the line of best fit. [3 marks / markah]
Plot melawan x, menggunakan skala 2 cm kepada 1 unit pada paksi-x dan 2 cm kepada 1 unit pada paksi- .
(c) Using the graph in 8(b), find the value of
Menggunakan graf di 8(b), cari nilai bagi
(i) h,
(ii) k. [5 marks / markah]
Answer Space

87
9 TRIAL STATE MRSM 2018
Use a graph paper to answer this question.
Guna kertas graf untuk menjawab soalan ini.
Table 9 shows the values of two variables, x and y obtained from an experiment. The variables x and y are
related by an equation y = , where p and q are constants.
Jadual 9 menunjukkan nilai-nilai bagi dua pembolehubah, x dan y yang diperolehi daripada suatu eksperimen.
Pembolehubah-pembolehubah x dan y dihubungkan oleh persamaan y = , dengan keadaan p dan q ialah
pemalar.
x 1 4 9 16 25 36
y 1.78 2.63 3.72 5.75 8.91 12.59
Table 9 / Jadual 9
(a) Based on Table 8, construct a table for the values of log10 y and . [2 marks / markah]
Berdasarkan pada Jadual 8, bina satu jadual untuk nilai-nilai bagi log10 y dan .
(b) Plot log10 y against , using a scale of 2 cm to 1 unit on -axis and 2 cm to 0.1 units on the log10 y-
axis. Hence, draw the line of best fit. [3 marks / markah]
Plot log10 y melawan , menggunakan skala 2 cm kepada 1 unit pada paksi- dan 2 cm kepada 0.1 units
pada paksi- log10 y . Seterusnya, lukis garis penyuaian terbaik.
(c) Using the graph in 9(b), find the value of
Menggunakan graf di 9(b), cari nilai bagi
(i) p,
(ii) q. [5 marks / markah]
Answer Space

88
10 TRIAL STATE KEDAH 2018
Use a graph paper to answer this question.
Guna kertas graf untuk menjawab soalan ini.
Table 10 shows the values of two variables, x and y obtained from an experiment. The variables x and y are
related by an equation = k + hx, where h and k are constants.
Jadual 10 menunjukkan nilai-nilai bagi dua pembolehubah, x dan y yang diperolehi daripada suatu eksperimen.
Pembolehubah-pembolehubah x dan y dihubungkan oleh persamaan = k + hx, dengan keadaan h dan k ialah
pemalar.
x 0.80 1.00 1.25 2.00 2.50 5.00
y 0.36 0.45 0.59 1.04 1.43 5.00
Table 10 / Jadual 10
(a) Based on Table 10, construct a table for the values of and . [2 marks / markah]
Berdasarkan pada Jadual 10, bina satu jadual untuk nilai-nilai dan .
(b) Plot against , using a scale of 2 cm to 0.2 units on -axis and 2 cm to 0.5 units on the -axis.
Hence, draw the line of best fit. [3 marks / markah]
Plot melawan , menggunakan skala 2 cm kepada 0.2 unit pada paksi- dan 2 cm kepada 0.5 units pada
paksi- . Seterusnya, lukis garis penyuaian terbaik .
(c) Using the graph in 10(b), find the value of
Menggunakan graf di 10(b), cari nilai bagi
(i) h,
(ii) k,
(iii) y when x = 1.6. [5 marks / markah]
y apabila x = 1.6.
Answer Space

89
FORM 5 CHAPTER 8 PROBABILITY DISTRIBUTIONS
SECTION B PAPER 2
1 SPM 2018 Paper 2 Q9
A study shows that the credit card balance of the customers is normally distributed as shown in Diagram 1.
Satu kajian menunjukkan hutang kad kredit bagi pelanggan-pelanggan adalah bertaburan normal seperti ditunjukkan
dalam Rajah 1.

Diagram 1 / Rajah 1
(a) (i) Find the standard deviation.
Cari sisihan piawai.
(ii) If 30 customers are chosen at random, find the number of customers who have a credit card
balance between RM1800 and RM3000.
Jika 30 pelanggan dipilih secara rawak, cari bilangan pelanggan yang mempunyai imbangan kad kredit
antara RM1800 dan RM3000. [7 marks / markah]
(b) It is found that 25% of the customers have a credit card balance less than RM y. Find the value of y.
Didapati bahawa 25% daripada pelanggan-pelanggan mempunyai hutang kad kredit kurang daripada RM y.
Cari nilai bagi y. [3 marks / markah]
Answer Space

90
2 SPM 2017 Paper 2 Q7
(a) The mass of honeydews produced in a plantation is normally distributed with a mean of 0.8 kg and a
standard deviation of 0.25 kg. The honeydew are being classified into three grades A, B and C according
to their masses:
Jisim bagi buah tembikai susu yang dihasilkan dalam tanaman adalah bertaburan normal dengan min 0.8 kg dan
sisihan piawai 0.25 kg. Tembikai susu itu dikelaskan kepada tiga gred A, B dan C mengikut jisim masing-masing:
Grade A > Grade B > Grade C
(i) The minimum mass of a grade A honeydew is 1.2 kg.
If a honeydew is picked at random from the plantation, find the probability that the honeydew is of
grade A.
Jisim minimum bagi gred A tembikai susu ialah 1.2 kg.
Jika sebiji tembikai susu dipilih secara rawak daripada tanaman itu, cari kebarangkalian bahawa tembikai
susu itu ialah red A.
(ii) Find the minimum mass, in kg, of grade B honeydew if 20% of the honeydews are of grade C.
Cari jisim minimum, dalam kg, bagi gred B tembikai susu jika 20% daripada tembikai susu tersebut ialah
gred C. [5 marks / markah]
(b) At the Shoot the Duck game booth at an amusement park, the probability of winning us 25%. Jason
bought tickets to play n games. The probability for Jason to win once is 10 times the probability of
losing all games.
Dalam permainan Menembak Itik di taman hiburan, kebarangkalian untuk menang ialah 25%. Jason telah
membeli tiket untuk bermain permainan itu sebanyak n kali. Kebarangkalian untuk Jason menang sekali dalam
permainan itu adalah 10 kali kebarangkalian kalah dalam semua permainan.
(i)Find the value of n.
Cari nilai bagi n.
(ii) Calculate the standard deviation of the number of wins.
Cari sisihan piawai bagi bilangan kemenangan. [5 marks / markah]
Answer Space

91
3 SPM 2016 Paper 2 Q9
(a) It is found that 20% of the students from Kampung Aman walk to school.
If 8 students from Kampung Aman are chosen at random, find the probability that exactly 3 of them
walk to school.
Didapati bahawa 20% daripada pelajar-pelajar dari Kampung Aman berjalan kaki ke sekolah.
Jika 8 pelajar dari Kampung Aman dipilih secara rawak, cari kebarangkalian bahawa tepat 3 orang daripada
mereka berjalan kaki ke sekolah. [2 marks / markah]
(b) The mass of pineapples harvested from a farm follows a normal distribution with a mean of 2 kg and a
standard deviation of m kg. It is given that 15.87% of the pineapples have a mass more than 2.5 kg.
Jisim nanas yang dituai daripada suatu ladang bertaburan normal dengan min 2 kg dan sisihan piawai m kg.
Diberi bahawa 15.87% daripada nanas itu mempunyai jisim lebih daripada 2.5 kg.
(i) Calculate the value of m.
Hitung nilai m.
(ii) Given the number of pineapples harvested from the farm is 1320, find the number of pineapples
that have the mass between 1.0 kg and 2.5 kg.
Diberi bilangan biji nanas yang dituai daripada ladang itu ialah 1320, cari bilangan nanas yang
mempunyai jisim antara 1.0 kg dan 2.5 kg. [8 marks / markah]
Answer Space

92
4 SPM 2015 Paper 2 Q10
(a) The probability of a student cycles to school is p. A sample of 5 students is selected at random.
Kebarangkalian seorang murid berbasikal ke sekolah ialah p. satu sampel 5 orang murid dipilih secara rawak.
(i) If the probability of all the 5 students cycle to school is 0.16807, find the value of p.
Jika kebarangkalian semua murid berbasikal ke sekolah ialah 0.16807, cari nilai bagi p.
(ii) Find the probability that more than 3 students cycle to school.
Cari kebarangkalian bahawa lebih daripada 3 orang murid berbasikal ke sekolah. [5 marks / markah]
(b) Diagram 4 shows a standard normal distribution graph representing the volume of chilli saunce in bottles
produced by a factory.
Rajah 4 menunjukkan satu graf taburan normal yang mewakili isipadu sos cili di dalam botol yang dihasilkan oleh
suatu kilang.

Diagram 4 / Rajah 4
It is given the mean is 950 cm3 and the variance is 256 cm6. If the percentage of the volume more than V
is 30.5%, find
Diberi bahawa min ialah 950 cm3 dan varians ialah 256 cm6. Jika peratus isipadu yang melebihi V ialah 30.5%,
cari
(i)the value of V,
nilai V,
(ii) the probability that the volume between 930 cm3 and 960 cm3.
kebarangkalian bahawa isipadu adalah antara 930 cm3 dan 960 cm3. [5 marks / markah]
Answer Space

93
5 SPM 2014 Paper 2 Q7
A survey is carried out about a scout in a school.
Suatu kajian dijalankan mengenai pengakap di sekolah .
(a) It is found that the mean of the number of scouts is 315, the variance is 126 and the probability that a
student participate in scout is p.
Didapati bahawa min untuk bilangan pengakap ialah 315, varians ialah 126 dan kebarngkalian bahawa seorang
murid menyertai pengakap ialah p.
(i) Find the value of p.
Cari nilai bagi p.
(ii) If 8 students from the school are chosen at random, find the probability that more than 5 students
participate in scout.
Jika 8 murid daripada sekolah itu dipilih secara rawak, cari kebarangkalia bahawa lebih daripada 5 murid
menyertai pengakap. [5 marks / markah]
(b) The mass of the scout members in the school follows a normal distribution with a mean of 48 kg and a
standard deviation of 5.8 kg. Find
Jisim bagi ahli pengakap di sekolah adalah mengikut taburan normal dengan min 48 kg dan sisihan piawai 5.8 kg.
Cari
(i) the probability that a member chosen at random from the group has a mass less than 45 kg.
kebarangkalian bahawa ahli yang dipilih daripada kumpulan itu mempunyai jisim kurang daripada 45 kg.
(ii)
the value of m, if 25% of the scout members have a mass more than m kg.
nilai m, jika 25% daripada ahli pengakap mempunyai jisim lebih daripada m kg. [5 marks / markah]
Answer Space

94
6 SPM 2013 Paper 2 Q10
(a) It is found that 70% of the students from a certain class obtained grade A in Geography in SPM trial
examination. If 10 students from the class are selected at random, find the probability that
Didapati bahawa 70% daripada murid dari suatu kelas memperolehi gred A dalam Geografi di peperiksaan
Percubaan SPM. Jika 10 murid daripada kelas itu dipilih secara rawak, cari kebarangkalian bahawa
(i) exactly 8 students obtained grade A,
tepat 8 murid memperolehi gred A,
(ii) not more than 8 students obtained grade A.
tidak lebih daripada 8 murid memperolehi gred A. [4 marks / markah]
(b) The Cumulative Grade Pointer Average (CGPA) of the final year students in a university following a
normal distribution with a mean of 2.6 and a standard deviation of 0.25.
Gred Purata Mata Terkumpul (CGPA) bagi pelajar tahun akhir dalam suatu universiti adalah mengikut
taburan normal dengan min 2.6 dan sisihan piawai 0.25.
(i) If one student is randomly selected, find the probability that the CGPA of the student is more than
3.0.
Jika seorang pelajar dipilih secara rawak, cari kebarangkalian bahawa CGPA bagi pelajar itu melebihi
3.0.
(ii) A degree will be given to the final year students who obtained CGPA more than k.
If 89.5% of the students manage to get a degree, find the value of k.
Satu ijazah akan diberi kepada pelajar tahun akhir yang memperolehi CGPA lebih daripada k.
Jika 89.5% daripada pelajar berjaya mendapat ijazah, cari nilai bagi k. [6 marks / markah]
Answer Space

95
7 TRIAL PAHANG JUJ 2019 Paper 2 Q9
(a) The probability of a voter walking to the polling station is p. A sample of 6 voters was randomly
selected.
Kebarangkalian seorang pengundi berjalan kaki ke stesen mengundi ialah p. Suatu sampel dengan 6 orang dipilih
secara rawak.
(i) If the probability of 6 voters walking to the polling center is 0.11765, find the value of p.
Jika kebarangkalian bahawa 6 orang berjalan kaki ke pusat mengundi ialah 0.11765, cari nilai bagi p.
(ii) Find the probability that at most two people are not walking.
Cari kebarangkalian bahawa selebih-lebihnya dua orang tidak berjalan kaki. [4 marks / markah]
(b) Diagram 7 shows a standard normal distribution graph representing the age group who came to vote in
the general election in a district.
Rajah 7 menunjukkan suatu graf taburan normal yang menggambarkan umur kumpulan yang datang mengundi di
suatu kawasan mengundi awam.

Diagram 7 / Rajah 7
It was found that the age of voters who came to the voting center was normally distributed with a mean
of 38.5 years and a standard deviation of 5.5 years. If the percentage of the age group in the shaded
region is 68.26%, find
Didapati bahawa umur pengundi yang datang ke pusat mengundi bertaburan normal dengan min 38.5 tahun dan
sisihan piawai 5.5 tahun. Jika peratus kumpulan umur di kawasan berlorek ialah 68.26%, cari
(i) the age range of the voters in the shaded region,
julat umur bagi pengundi di kawasan berlorek,
(ii) the probability that age groups who come to a vote exceed 55 years.
kebarangkalian bahawa kumpulan umur yang datang mengundi melebih 55 tahun. [6 marks / markah]
Answer Space

96
9 (a) 40% of the students in a school go to school by bus. A sample of 10 students is chosen. Find the
probability that
40% daripada pelajar di sekolah pergi ke sekolah dengan bs. Satu sampel 10 orang pelajar dipilih. Cari
kebarangkalian bahawa
(i) 3 of them go to school by bus,
3 daripada mereka pergi ke sekolah dengan bas,
(ii) more than 7 students go to school by bus.
lebih daripada 7 orang pelajar pergi ke sekolah dengan bas. [4 marks / markah]
(b) The lifespan of a car tyre from a factory is normally distributed with a mean of 2 years and a standard
deviation of 3 months. Find the probability that a tyre chosen at random form the factory will last for
Jangka hayat bagi sebiji tayar kereta dari kilang ialah bertaburan normal dengan min 2 tahun dan sisihan piawai
3 bulan. Cari kebarangkalian bahawa sebiji tayar yang dipilih secara rawak daripada kilang akan bertahan untuk
(i) at least 2.5 years,
sekurang-kurangnya 2.5 tahun,
(ii) 1.5 years to 2 years 2 months.
1.5 tahun hingga 2 tahun 2 bulan. [6 marks / markah]
Answer Space

97
FORM 5 CHAPTER 10 LINEAR PROGRAMMIG
PAPER 2 SECTION C
1 Translating the sentences to inequalities
Sentence Inequality
The total x and y is at least 20 x + y  20
Jumlah x dan y mesti sekurang-kurangnya 20
The total x and y is at most 20 x + y  20
Jumlah x dan y mesti selebih-lebihnya 20
The total x and y is more than 20 x + y > 20
Jumlah x dan y mesti lebih daripada 20
The total x and y is less than 20 x + y < 20
Jumlah x dan y kurang daripada 20
The total x and y is not less than 20 x + y  20
Jumlah x dan y tidak kurang daripada 20
The total x and y is not more than 20 x + y  20
Jumlah x dan y tidak melebihi 20
x must be at least twice the y x  2y
x mesti sekurang-kurangnya dua kali y
x must be at most twice the y x  2y
x mesti selebih-lebihnya dua kali y
x must exceed y at most 20 x – y  20
x mesti melelbih y dengan selebih-lebihya 20
x must exceed y at least 20 x – y  20
x mesti melebihi y dengan sekurang-kurangnya 20
x must exceed y with more than 20 x – y > 20
x mesti melebihi y dengan lebih daripada 20
Ratio of x to the y is at least with 3 : 2 x:y3:2 Or
Nisbah x kepada y sekurang-kurangnya 3 : 2  
2x  3y x y
3y  2y
yx
Ratio of x to the y is at most with 3 : 2 x:y3:2 Or
Nisbah x kepada y selebih-lebihnya 3 : 2  
2x  3y x y
3y  2y
yx
x is at most 80% of the y x y
x selebih-lebihnya 80% daripada y
100x  80y
80y  100x
x is at least 75% of the y x y
x sekurang-kurangnya 75% daripada y
100x  75y
75y  100x

Example: For Question (b)(ii)


Maximum cost = 30x + 40y
Let 30x + 40y = 120, 240, 360, …….1200, 2400, 3600,…

Multiple of 30 and 40. Maybe 120 or 240 or 360 or……depends on
the scale of axes.

Line
 and  : Solid line

> and < : Dashed-line

Shaded Region R
Make sure to arrange the inequalities so that the y is at the left side before
determining the region R.
Let say you get the inequality x  3y
Rearrange: 3y  x  Shade above the line of 3y = x

98
1 SPM 2018 Paper 2 Q15
Use the graph paper provided to answer this question. Detach the graph paper and tie it together with your answer
booklet.
Guna kertas graf yang disediakan untuk menjawab soalan ini. Kepilkan kertas graf dan ikat bersama dengan buku jawapan anda.
A department store sells two types of perfumes, P and Q. The selling price of perfume P is RM400 per bottle and
perfume Q is RM100 per bottle. A promoter, Zie sells x bottles of perfume P and y bottles of perfume Q. She will be
given a 5% commission of her total sales if she is able to achieve the following targets:
Sebuah pusat membeli-belah menjual dua jenis pewangi, P dan Q. Harga jualan bagi pewangi P is RM400 sebotol dan pewangi Q
ialah RM100 sebotol. Seorang promoter, Zie menjual x botol pewangi P dan y botol pewangi Q. Dia akan diberi komisen 5%
daripada jumlah jualannya jika dia mampu mencapai sasaran berikut :
I The total number of bottles of perfume sold is at least 50.
Jumlah bilangan botol pewangi yang dijual mesti sekurang-kurangnya 50.
II The minimum total sale is RM8000.
Jumlah jualan minimum ialah RM8000.
(a) Write two inequalities, other than x ≥ 0 and y ≥ 0, which satisfy all the above constraints. [2 marks / markah]
Tulis dua ketaksamaan, selain daripada x ≥ 0 dan y ≥ 0, yang memuaskan kesemua kekangan di atas.
(b) Use a scale of 2 cm to 10 bottles on both axes, construct and shade the region R which satisfies all the above
constraints.
Menggunakan skala 2 cm kepada 10 botol pada kedua-dua paksi, bina dan lorekkan rantau R yang memuskan kesemua
kekangan. [3 marks / markah]
(c) Use the graph constructed in 1(b) to answer the following questions:
Guna graf yang dibina di 1(b) untuk menjawab soalan berikut:
(i) Find the minimum number of bottles of perfume P and perfume Q to be sold if the number of bottles of
perfume Q sold is twice the number of bottles of perfume P.
Cari bilangan minimum botol pewangi P dan pewangi Q yang akan dijual jika bilangan botol pewangi Q yang dijual
ialah dua kali bilangan botol pewangi P.
(ii)Determine the minimum amount of commissions received by Zie if she is able to sell 35 bottles of perfume P.
Tentukan jumlah minimum komisen yang diterima oleh Zie jika dia mampu menjual 35 bottles pewangi P.
[5 marks / markah]
Answer Space
(a) I : x + y  50
II : 400x + 100y  8000

(b) For x + y  50 For 400x + 100y  8000


Let x + y  50 Let 400x + 100y  8000
x 0 50 x 0 20
y 50 0 y 80 0
On graph paper [Remember to label the axes, the lines]

(c)(i) y = 2x
x 0 20
y 0 40
From graph, (17, 34) is the minimum point.
So, number of perfume P sold is 17 and the number of perfume Q sold is 34

(c)(ii) From graph, if perfume P, x = 35, then perfume Q, y = 15


So, total commissions = [35(RM400) + 15(RM100) ] ×
= RM775

99
Graph of 1(b)
Question sine i against sine r
Graf sin i melawan sin r

y
100

90

80

400x + 100y = 8000


70

y = 2x
60

50

R
40

(17, 34)
30

20

10
x + y = 50

0 10 20 30 x = 35 40 50 60 70 80 x

100
2 SPM 2017 Paper 2 Q13
Use the graph paper provided to answer this question. Detach the graph paper and tie it together with your answer
booklet.
Guna kertas graf yang disediakan untuk menjawab soalan ini. Kepilkan kertas graf dan ikat bersama dengan buku jawapan anda.
An entrepreneur wants to produce two types of toys, P and Q by using a machine. In a day, the machine produces x
number of type P toys and y number of type Q toys. The time required to produce a type of P toy is 6 minutes and the
time required to produce a type of Q toy is 5 minutes. The production of the toys is based on the following constraints:
Seorang peniaga ingin menghasilkan dua jenis alat mainan, P dan Q dengan menggunakan mesin. Pada suatu hari, mesin itu
menghasilkan x jenis alat mainan P dan y jenis alat mainan Q. Masa yang diperlukan untuk menghasilkan alat mainan P ialah 6 minit
dan masa yang diperlukan untuk menghasilkan alat mainan Q ialah 5 minit. Penghasilan alat mainan adalah berdasarkan pada
kekangan berikut:
I The total number of toys produced must be more than 40 units in a day.
Jumlah bilangan alat mainan yang dihasilkan mesti melebihi 40 unit sehari.
II The machine can operate for only 15 hours a day.
Mesin itu boleh beroperasi selama 15 jam sahaja dalam sehari.
III The ratio of the number of type P toys to the number of type Q toys is at most 3 : 5.
Nisbah bilangan jenis alat mainan P kepada bilangan alat mainan Q ialah selebih-lebihnya 3 : 5.
(a) Write two inequalities, other than x ≥ 0 and y ≥ 0, which satisfy all the above constraints. [3 marks / markah]
Tulis dua ketaksamaan, selain daripada x ≥ 0 dan y ≥ 0, yang memuaskan kesemua kekangan di atas.
(b) Use a scale of 2 cm to 20 toys on both axes, construct and shade the region R which satisfies all the above
constraints. [3 marks / markah]
Menggunakan skala 2 cm kepada 20 alat mainan pada kedua-dua paksi, bina dan lorekkan rantau R yang memuskan kesemua
kekangan
(c) Use the graph constructed in 2(b), find the range of total sales that can be obtained if the selling price of a type P
toy is RM5 and the selling price of a type Q toy is RM3. [4 marks / markah]
Menggunakan graf yang dibina di 2(b), cari julat jumlah jualan yang boleh diperolehi jika harga jualan jenis alat mainan P
ialah RM5 dan harga jualan alat mainan jenis Q ialah RM3.
Answer Space
(a) I : x + y > 40
II : 6x + 5y  15 × 60
6x + 5y  900
III : x : y  3 : 5

x y
y x

(b) For x + y > 40 For 6x + 5y  900 For y  x


Let x + y = 40 For 6x + 5y = 900 For y = x
x 0 40 x 0 150 x 0 30
y 40 0 y 180 0 y 0 50
On graph paper

(c) Total sales = 5x + 3y


Let 5x + 3y = 150  pilih 150 atau 60 or apa saja nombor asalkan ialah gandaan kedua-dua 5 dan 3
x 0 30
y 50 0

Minimum sales is at (0, 40). The sales = 5(0) + 3(40)


= 120

Maximum sales is at (62, 105). The sales = 5(62) + 3(105)


= 625

So, the range of total sales is RM120 < Total sale  RM625

101
Graph of 2(b)
Question sine i against sine r
Graf sin i melawan sin r

y
200

180

6x + 5y = 900

160

140

120 y = 5/3 x

R (62, 105) Maximum point


100

80

60

40
(0, 40) Minimum point

20
x + y = 40

0 20 40 60 80 100 120 140 160 x


5x + 3y = 150

102
3 SPM 2014 Paper 2 Q
Use the graph paper provided to answer this question. Detach the graph paper and tie it together with your answer
booklet.
Guna kertas graf yang disediakan untuk menjawab soalan ini. Kepilkan kertas graf dan ikat bersama dengan buku jawapan anda.
A company wants to buy two types of tables, A and B for its canteen. The price of type A table is RM200 and a type B table is RM100.
The area of a table top of type A is 1 m2 and a type B table is 2 m2. The company buys x tables of type A and y tables of type B. The
purchase of the table is based on the following constraints:
Suatu syarikat ingin membeli dua jenis meja, A dan M untuk kegunaan kantinnya. Harga untuk meja jenis A ialah RM200 dan meja
jenis B ialah RM100. Luas bagi meja jenis A ialah 1 m2 dan meja jenis B ialah 2 m2. Syarikat itu membeli x jenis meja A dan y jenis
meja B. Pembelian meja adalah berdasarkan pada kekangan berikut:
I The total area of the tables is not less than 30m2.
Jumlah luas meja tidak boleh kurang daripada 30m2.
II The amount of money allocated is RM6000.
Jumlah peruntukan kewangan ialah RM6000.
III The number of type B table is at most two times the number of type A table.
Bilangan meja jenis B selebih-lebihnya dua kali bilangan jenis meja A.
(a) Write three inequalities, other than x ≥ 0 and y ≥ 0, which satisfy all the above constraints. [3 marks / markah]
Tulis tiga ketaksamaan, selain daripada x ≥ 0 dan y ≥ 0, yang memuaskan ketiga-tiga kekangan di atas.
(b) Using a scale of 2 cm to 5 tables on both axes, construct and shade the region R which satisfies all the above
constraints. [3 marks / markah]
Menggunakan skala 2 cm kepada 5 meja pada kedua-dua paksi, bina dan lorek rantau R yang memuaskan kesemua kekangan di
atas.
(c) Using the graph constructed in 3(b), find
Menggunakan graf yang dibina di 3(b), cari
(i) the range of the number of type A table if 10 of type B tables are bought,
julat bilangan meja jenis A jika 10 jenis meja B dibeli,
(ii) the maximum number of customers that can use the tables at a time if a type A table can accommodate 4
customers and a type B table can accommodate 8 customers. [4 marks / markah]
bilangan maksimum pelanggan yang boleh menggunakan meja sepanjang masa jika meja jenis A boleh memenuhi 4
pelanggan dan meja jenis B boleh memenuhi 8 pelanggan.
Answer Space

103
4 SPM 2013 Paper 2 Q
Use the graph paper provided to answer this question. Detach the graph paper and tie it together with your answer
booklet.
Guna kertas graf yang disediakan untuk menjawab soalan ini. Kepilkan kertas graf dan ikat bersama dengan buku jawapan anda.
A tourist boat operator provide x trips to Island A and y trips to Island B per day. The service provided is based on the
following constraints:
Satu operasi perahu pelancong menyediakan x perjalanan ke Pulau A dan y perjalanan ke Pulau B dalam satu hari. Perkhidmatan
yang disediakan adalah berdasarkan pada kekangan berikut :
I The total number of trips provided is not more than 16.
Jumlah bilangan perjalanan yang disediakan tidak boleh melebihi 16.
II The number of trips to Island A is not more than three times the number of trips to Island B.
Bilangan perjalanan ke Pulau A tidak boleh melebihi tiga kali bilangan perjalanan ke Pulau B.
III The fare per trip to Island A is RM60 and the fare per trip to Island B is RM30. The total fare collection per day is
not less than RM540.
Tambang untuk setiap perjalanan ke Pulau A ialah RM60 dan tambang untuk setiap perjalanan ke B ialah RM30. Jumlah
tambang yang dikutip dalam satu hari tidak boleh kurang daripada 540.
(a) Write three inequalities, other than x ≥ 0 and y ≥ 0, which satisfy all the above constraints. [3 marks / markah]
Tulis tiga ketaksamaan, selain daripada x ≥ 0 dan y ≥ 0, yang memuaskan kesemua kekangan di atas.
(b) Using a scale of 2 cm to 2 trips on both axes, construct and shade the region R which satisfies all of the above
constraints. [3 marks / markah]
Menggunakan skala 2 cm kepada 2 perjalanan untuk kedua-dua paksi, bina dan lorek rantau R yang memuaskan kesemua
kekangan di atas.
(c) Using the graph constructed in 4(b), find
Menggunakan graf yang dibina di 4(b), cari
(i) the minimum number of trips to Island B if 8 trips to Island A are provided per day.
bilangan minimum perjalanan ke Pulau B jika 8 perjalanan ke Pulau A disediakan dalam sehari.
(ii) the maximum total profit per day if the profit per trip to Island A is RM40 and the profit per trip to Island B is
RM20. [4 marks / markah]
maksimum keuntungan dalam sehari jika keuntungan bagi satu perjalanan ke Pulau A ialah RM40 dan keuntungan bagi
satu perjalanan ke Pulau B ialah RM20.
Answer Space

104
5 SPM 2012 Paper 2 Q
Use the graph paper provided to answer this question. Detach the graph paper and tie it together with your answer
booklet.
Guna kertas graf yang disediakan untuk menjawab soalan ini. Kepilkan kertas graf dan ikat bersama dengan buku jawapan anda.
A carpenter makes two types of furniture, table and chair. In a week, he makes x tables and y chairs. He has a capital of
RM6000. The production of furniture is based on the following constraints:
Seorang tukang kayu membuat dua jenis perabot, meja dan kerusi. Dalam satu minggu, dia membuat x meja dan y kerusi. Dia
mempunyai modal RM6000. Penghasilan perabot adalah berdasarkan kepada kekangan berikut :
I The cost of making a table is RM80 and a chair is RM60.
Kos membuat satu meja ialah RM80 dan satu kerusi ialah RM60.
II The minimum total number of tables and chairs must be 50.
Jumlah bilangan minimum meja dan kerusi mesti 50.
III The number of chairs must be at least 80% of the number of tables.
Bilangan kerusi mesti sekurang-kurangnya 80% daripada bilangan meja.
(a) Write three inequalities, other than x ≥ 0 and y ≥ 0, which satisfy all the above constraints. [3 marks / markah]
Tulis tiga ketaksamaan, selain daripada x ≥ 0 dan y ≥ 0, yang memuaskan kesemua kekangan di atas.
(b) Using a scale of 2 cm to 10 furniture on both axes, construct and shade the region R which satisfies all of the above
constraints. [3 marks / markah]
Menggunakan skala 2 cm kepada 10 perabot untuk kedua-dua paksi, bina dan lorek rantau R yang memuaskan kesemua
kekangan di atas.
(c) Using the graph constructed in 5(b), find
Menggunakan graf yang dibina di 5(b), cari
(i) the minimum number of chairs made if 24 tables are made,
bilangan minimum kerusi yang dibuat jika 24 meja telah dibuat,
(ii) the maximum total profit obtained if the profit for a table is RM40 and the profit for a chair is RM20.
jumlah keuntungan maksimum yag diperolehi jika keuntungan untuk sebuah meja ialah RM40 dan keuntungan untuk
sebuah kerusi ialah RM20. [4 marks / markah]
Answer Space

105
6 SPM 2011 Paper 2 Q
Use the graph paper provided to answer this question. Detach the graph paper and tie it together with your answer
booklet.
Guna kertas graf yang disediakan untuk menjawab soalan ini. Kepilkan kertas graf dan ikat bersama dengan buku jawapan anda.

A company obtained a painting tender of a government building. The building is to be painted with undercoat paint and
gloss paint. The number of tins of the undercoat paint is x and the number of tins of the gloss paint is y. The painting
works based on the following constraints:
Sebuah syarikat mendapat projek menecat bangunan kerajaan. Bangunan itu akan dicat dengan menggunakan cat lapis dan cat kilat.
Bilangan tin cat lapis ialah x dan bilangan tin cat kilat ialah y. Kerja mengecat adalah berdasarkan kepada kekangan berikut :
I The number of tins of the gloss paint is not more than 2 times the number of tins of the undercoat paint.
Bilangan tin cat kilat tidak boleh melebih dua kali bilangan tin cat lapis.
II The number of tins of the gloss paint is at least ¼ of the number of tins of the undercoat paint.
Bilangan tin cat kilat mestilah sekurang-kurangnya ¼ daripada bilangan tin cat lapis.
III The total time allocated for the painting work is at most 120 hours. For a tin of undercoat paint, the painting time is
3 hours while for a tin of gloss paint, the painting time is 2 hours.
Jumlah masa yang diperuntukkan ialah selebih-lebihnya 120 jam. Untuk satu tin cat lapis, masa mengecat ialah 3 jam
manakala untuk satu tin cat kilat, masa mengecat ialah 2 jam.
(a) Write three inequalities, other than x ≥ 0 and y ≥ 0, which satisfy all the above constraints. [3 marks / markah]
Tulis tiga ketaksamaan, selain daripada x ≥ 0 dan y ≥ 0, yang memuaskan kesemua kekangan di atas.
(b) Using a scale of 2 cm to 5 tins on both axes, construct and shade the region R which satisfies all of the above
constraints. [3 marks / markah]
Menggunakan skala 2 cm kepada 5 tin untuk kedua-dua paksi, bina dan lorek rantau R yang memuaskan kesemua kekangan di
atas.
(c) Using the graph constructed in 6(b), find
Menggunakan graf yang dibina di 6(b), cari
(i) the minimum number of tins of the gloss paint if the number of tins of the undercoat paint is 30.
bilangan tin minimum cat kilat jika bilangan tin cat lapis ialah 30.
(ii) the maximum expenditure for the paint if the price of the undercoat paint is RM50 per tin and the price of
the gloss paint is RM70 per tin. [4 marks / markah]
perbelanjaan maksimum untuk cat jika harga cat lapis ialah RM50 satu tin dan harga untuk cat kilat ialah RM70 satu
tin.
Answer Space

106
7 Use the graph paper provided to answer this question. Detach the graph paper and tie it together with your answer
booklet.
Guna kertas graf yang disediakan untuk menjawab soalan ini. Kepilkan kertas graf dan ikat bersama dengan buku jawapan anda.
A tailor makes two types of clothes, trousers and shirt. In a week, he makes x trousers and y shirts. The cost of making a
trouser is RM80 and a shirt is RM40. The production of clothes is based on the following constraints:
Seorang penjahit membuat dua jenis pakaian iaitu seluar dan baju. Dalam seminggu, dia membuat x seluar dan y baju
Kos membuat seluar ialah RM80 dan membuat baju ialah RM40. Penghasilan baju-baju adalah berdasarkan pada
kekangan berikut:
I The maximum total number of trousers and shirts must be 80.
Jumlah maksimum seluar dan baju mestilah 80.
II The number of shirt must exceed the number of trousers by at least 5.
Bilangan baju mesti melebihi bilangan seluar sekurang-kurangnya 5.
III The minimum cost to produce the clothes is RM3200.
Kos minimum menghasilkan baju ialah RM3200.
(a) Write three inequalities, other than x  0 and y  0 which satisfy all the above constraints. [3 marks / markah]
Tulis tiga ketaksamaan, selain daripada x ≥ 0 dan y ≥ 0, yang memuaskan kesemua kekangan di atas.
(b) Using a scale of 2 cm to 10 clothes on both axes, construct and shade the region R which satisfies all the above
constraints.
Menggunakan skala 2 cm kepada 10 baju untuk kedua-dua paksi, bina dan lorek rantau R yang memuaskan kesemua kekangan
di atas [3 marks / markah]
(c) Using the graph constructed in 7(b), find
Menggunakan graf yang dibina di 7(b), cari
(i) the minimum number of shirts produced.,
bilangan minimum baju yang dihasilkan,
(ii) the maximum cost for the production of the clothes. [4 marks / markah]
kos maksimum untuk penghasilan baju itu.
Answer Space

107
8 Use the graph paper provided to answer this question. Detach the graph paper and tie it together with your answer
booklet.
Guna kertas graf yang disediakan untuk menjawab soalan ini. Kepilkan kertas graf dan ikat bersama dengan buku jawapan anda.
A prestige college offers two courses, A and B. The enrolment of students is based on the following constraints:
Suatu kolej menawarkan dua jenis kursus, A dan B. Pendaftaran pelajar adalah berdasarkan kepada kekangan berikut :
I: The capacity of the college is 170 students.
Kapasiti bagi kolej itu ialah 170 pelajar.
II: The minimum total number of students enrolled is 80.
Jumlah minimum bagi murid yang didaftarkan ialah 80.
III: The number of students enrolled for course B exceeds twice the number of students enrolled for course A by at least
20 students.
Bilangan murid yang mendaftar untuk kursus B melebihi dua kali bilangan murid yang mendaftar untuk kursus A
sekurang-kurangnya 20.
Given that there are x students enrolled for course A and y students enrolled for course B.
Diberi ia mempunyai x pelajar yang mendaftar untuk kursus A dan y pelajar yang mendaftar untuk kursus B .
(a) Write three inequalities, other than x  0 and y  0, that satisfy all the above constraints. [3 marks / markah]
Tulis tiga ketaksamaan, selain daripada x ≥ 0 dan y ≥ 0, yang memuaskan kesemua kekangan di atas.
(b) Using a scale of 2 cm to 10 students on the x-axis and 2 cm to 20 students on the y-axis, construct and shade the
region R which satisfies all the above constraints. [3 marks / markah]
Menggunakan skala 2 cm kepada 10 pelajar pada paksi-x dan 2 cm kepada 20 pelajar pada paksi-y, bina dan lorek rantau R
yang memuaskan kesemua kekangan di atas.
(c) Using the graph constructed in 8(b), find
Menggunakan graf yang dibina di 8(b), cari
(i) the maximum amount of fees collected per month if the monthly fees for course A is RM100 and for course
B is RM80.
jumlah maksimum yuran yang dikutip setiap bulan jika yuran bulanan untuk kursus A ialah RM100 dan untuk kursus B
ialah RM80.
the range of the number of students enrolled for course B if the number of students enrolled for course A is
(ii)
30. [4 marks / markah]
julat bilangan murid yang mendaftar untuk kursus B jika bilangan murid yang mendaftar untuk kursus A ialah 30.
Answer Space

108
FORM 4 CHAPTER 11 INDEX NUMBER
PAPER 2 SECTION C
1 Table 1 shows the prices and the price indices for the four ingredients K, L, M, and N, used in making a particular kind of
bread. Diagram 1 is a pie chart which represents the ratio of the amount of the ingredients K, L, M and N used in making
the bread.
Jadual 1 menunjukkan harga dan indeks harga bagi empat bahan, K, L, M dan N, yang digunakan untuk membuat sejenis roti. Rajah
1 ialah satu carta pai yang mewakili nisbah kuantiti bahan K, L, M dan N yang digunakan untuk membuat roti itu.

Ingredients Price / Harga (RM / kg) Price index for the year 2006 based on
Bahan Year / Tahun the year 2003
N K
Indeks harga pada tahun 2006 berasaskan
2003 2006 tahun 2003 80o
K 1.60 2.00 p 100o
L 4.00 q 120 120o L
M 0.80 1.20 150 M
N r 1.60 80
Table 1 / Jadual 1 Diagram 1 / Rajah 1

(a) Find the values of p, q and r. [3 marks]


Cari nilai p, q dan r.
Solution
Recent Price
Remember the formulae: Price index, P = × 100
Previous Price
For p: For q: For r:
2.00 q 1.60
p= × 100 120 = × 100 80 = × 100
1.60 4 R
120  4 1.60
p = 125 q= r= × 100
100 80
q = 4.80 r = 2.00

So, the p = 125, q = RM 4.80 and r = RM 2.00

(b) (i) Calculate the composite index for the cost of making the bread in the year 2006 based on the year 2003.
Hitung indeks gubahan bagi kos untuk membuat roti itu pada tahun 2006 berasaskan tahun 2003. [3 marks]
Solution

Remember the formulae: Composite index, I =


I W i i Total (Price index  weightage)
W
or
i
Total weightage

[125  60  120  100  150  120  80  80]


Composite index, I =
[60  100  120  80]
43900
=
360
= 121.94

(ii) Hence, calculate the cost of making the bread in the year 2003 if the corresponding cost in the year 2006 is
RM 40. [2 marks]
Oleh yang demikian, hitung kos untuk membuat roti itu pada tahun 2003 jika kos sepadan pada tahun 2006 ialah RM40.
Solution

Cost (Recent)
Remember to calculate the cost of making → Use composite index = × 100
Cost (Previous)
Cost in Year 2006
Composite index = × 100
Cost in Year 2003
RM40
121.94 = × 100
Cost in Year 2003
RM40
Cost in Year 2003 = × 100
121.94
= RM 32.80

109
(c) The cost of making the bread is expected to increase by 40% from the year 2006 to the year 2010.
Find the composite index for the cost of making the bread in the year 2010 based on the year 2003. [2 marks]
Kos untuk membuat roti itu dijangka meningkat sebanyak 40% dari tahun 2006 ke tahun 2010.
Cari indeks gubahan bagi kos membuat roti itu pada tahun 2010 berasaskan tahun 2003.
Solution
Cost of making in Year 2003 is RM 32.80
Cost of making in Year 2006 is RM 40.00
From Year 2006 to the Year 2010, it has been increased by 40%
140
So, the cost of making in Year 2010 is = RM 40.00 ×
100
= RM 56.00
Cost (Year 2010)
So, the new composite index = × 100
Cost (Year 2003)
RM 56.00
= × 100
RM 32.80
= 170.73

EXERCISE
1 Table 1 shows the price indices and percentage of usage of four main ingredients, P, Q, R and S, which are used to make
a type of cake.
Jadual 1 menunjukkan indeks harga dan peratus penggunaan empat bahan utama, P, Q, R dan S, yang digunakan untuk membuat satu
jenis kek.
Ingredients Price index for the year 2007 based on the year 2005 Percentage of usage
Bahan Indeks harga pada tahun 2007 berasaskan tahun 2005 Peratus penggunaan
P 145 20
Q 120 30
R 115 40
S x 10
Table 1 / Jadual 1
(a) Calculate / Hitung
(i) the price of P in the year 2005 if the price in the year 2007 is RM 26.10. [2 marks]
harga P pada tahun 2005 jika harganya pada tahun 2007 ialah RM 26.10. [Ans: 18]
(ii) the price index of Q in the year 2007 based on the year 2003 if its price index in the year 2005 based on the
year 2003 is 145. [3 marks]
indeks harga bagi Q pada tahun 2007 berasaskan tahun 2003 jika indeks harganya pada tahun 2005 berasaskan tahun
2003 ialah 145. [Ans: 82.76]
(b) The composite index for the cost of making the cake in the year 2007 based on the year 2005 is 125.
Indeks gubahan bagi kos membuat kek itu pada tahun 2007 berasaskan tahun 2005 ialah 125.
Calculate / Hitung
(i) the value of x, [3 marks]
nilai x, [Ans: 140]
(ii) the cost of making a cake in the year 2005 if the corresponding cost in the year 2007 is RM 72.
kos untuk membuat sebiji kek itu pada tahun 2005 jika kos sepadan pada tahun 2007 ialah RM72. [2 marks][Ans: 57.6]
Answer space

110
2 Table 2 shows the prices and the price indices of five components, P, Q, R, S and T, used to produce a type of toy car.
Diagram 2 shows a pie chart which represents the ratio of the quantity of components used.
Jadual 2 menunjukkan harga dan indeks harga bagi lima komponen, P, Q, R, S dan T, yang digunakan untuk membuat satu jenis
kereta mainan. Rajah 2 menunjukkan satu carta pai yang mewakili nisbah kuantiti bagi komponen yang digunakan.
Price(RM) in the year
Component Harga(RM) pada tahun
Price index for the year 2007 based on the year 2005
Komponen Indeks harga pada tahun 2007 berasaskan tahun 2005
2005 2007
P m 4.40 110
Q 4.00 5.60 140
R 2.40 3.00 125
S 6.00 5.40 n
T 8.00 12.00 150
Table 2 / Jadual 2

144o Q
36o
S
72o
P
T

Diagram 2 / Rajah 2

(a) Find the value of m and of n. [3 marks]


Cari nilai m dan nilai n. [Ans: m = 4, n = 90]
(b) Calculate the composite index for the cost of making the toy car in the year 2007 based on the year 2005.
Hitung indeks gubahan bagi kos untuk membuat kereta mainan itu pada tahun 2007 berasaskan tahun 2005.
[3 marks] [Ans: 126]
(c) The price of each component increases by 25% from the year 2007 to the year 2009. Given that the production cost
of one toy car in the year 2005 is RM60, calculate the corresponding cost in the year 2009.
Harga setiap komponen meningkat sebanyak 25% dari tahun 2007 ke tahun 2009. Diberi kos untuk membuat sebuah kereta
mainan itu pada tahun 2005 ialah RM60, hitung kos sepadan pada tahun 2009.
[4 marks] [Ans: 94.5]
Answer space

111
3 Table 3 shows the price indices for the year 2013 based on the year 2010, changes in price indices from the year 2013 to
the year 2014 and the weightages of four brands of sports shoes.
Jadual 3 menunjukkan indeks harga pada tahun 2013 berasaskan tahun 2010, perubahan indeks harga dari tahun 2013 ke tahun 2014
dan pemberat bagi empat jenama kasut sukan.
Brand of sports shoe Price index Change in price index Weightage
Jenama kasut sukan Indeks harga Perubahan indeks harga Pemberat
(2013 based 2010) (From 2013 to 2014)
P 128 Decrease 5 % 3
Menyusut 5 %
Q 130 No change 4
Tiada perubahan
R 105 No change x
Tiada perubahan
S 145 Increase 15 % 2
Menokok 15 %
Table 3 / Jadual 3
(a) The price of brand P and Q sports shoe is RM 40 and RM 50 respectively in the year 2010.
Ali buys a pair of brand P and 2 pairs of brand Q sport shoes in 2013.
Calculate the amount Ali has to pay in that year. [3 marks]
Harga sepasang kasut sukan jenama P dan Q masing-masing ialah RM40 dan RM50 sepasang pada tahun 2010.
Ali membeli sepasang kasut jenama P dan 2 pasang kasut jenama Q pada tahun 2013.
Hitung amaun yang Ali perlu bayar pada tahun tersebut. [Ans: 181.20]
(b) The composite index for the prices of the four brands of sports shoe in the year 2013 based on the year 2010 is
125.75.
Indeks gubahan bagi harga empat jenama kasut sukan itu pada tahun 2013 berasaskan tahun 2010 ialah 125.75.
(i) Calculate the value of x. [2 marks]
Hitung nilai x. [Ans: 3]
(ii) Hence, calculate the composite index for the prices of the four brands of sports shoe in the year 2014 based
on the year 2010.
Oleh yang demikian, hitung indeks gubahan bagi harga empat jenama kasut sukan itu pada tahun 2014 berasaskan
tahun 2010. [3 marks] [Ans: 127.775]
(iii) Calculate the price of a pair of sports shoe in the year 2014 if the corresponding price in the year 2010 is
RM75.
Hitung harga sepasang kasut sukan pada tahun 2014 jika harga sepadan pada tahun 2010 ialah RM75.
[2 marks] [Ans: 95.83]
Answer space

112
INTENSIVE EXERCISE
CHAPTER 11 INDEX NUMBERS
1 SPM 2018 Paper 2 Q13
Table 1 shows the information related to four ingredients, P, Q, R and S used in the production of a type of noodle.
Jadual 1 menunukkan makulat yang berkaitan dengan empat jenis ramuan, P, Q, R dan S yang digunakan dalam penghasilan mi .
Ingredient Change in price index from the year 2013 to the year 2017 Percentage of usage (%)
Ramuan Perubahan indeks harga dari tahun 2013 ke tahun 2017 Peratus penggunaan (%)
P 40% increase / bertambah 10
Q 20% increase / bertambah 10
R 60% increase / bertambah
S 10% decrease / bertambah 50
Table 1 / Jadual 1
The production cost for this noodle is RM47 600 in the year 2017.
Kos penghasilan untuk mi ini ialah RM47 600 dalam tahun 2017.
(a) If the price of ingredient Q in the year 2013 is RM4.20, find its price in the year 2017. [2 marks / markah]
Jika harga ramuan Q dalam tahun 2013 ialah RM4.20, cari harganya dalam tahun 2017.
(b) Percentage of usage for several ingredients were given in Table 1.
Calculate the corresponding production cost in the year 2013. [5 marks / markah]
Peratus penggunaan untuk beberapa ramuan diberi di Jadual 1.
Hitung kos penghasilan yang sepadan dalam tahun 2013.
(c) The production cost is expected to increase by 50% from the year 2017 to the year 2019. Calculate the percentage
of changes in production cost from the year 2013 to the year 2019. [3 marks / markah]
Kos penghasilan dijangka akan menaik sebanyak 50% dari tahun 2017 ke tahun 2019. Hitung peratus perubahan dalam kos
penghasilan dari tahun 2013 ke tahun 2019.
Answer Space

113
2 SPM 2017 Paper 2 Q13
Table 2 shows the prices and price indices of three types of ingredients A, B and C used in the production of a type of a
fish ball.
Jadual 2 menunjukkan harga dan indeks harga bagi tiga jenis bahan, A, B dan C yang digunakan dalam penghasilan suatu jenis
bebola ikan.
Price index for the year 2016
Price (RM) per kg for the year based on the year 2014 Weightage
Ingredient
Harga (RM) per kg untuk tahun Indeks harga untuk tahun 2016 Pemberat
Bahan berdasarkan tahun 2014
2014 2016
A 5.00 6.64 132.8 50
B y 3.00 x 20
C 0.50 0.95 190 1
Table 2 / Jadual 2
(a) The price of ingredient B is increased by 20% from the year 2014 to the year 2016.
Harga bagi bahan B bertambah sebanyak 20% dari tahun 2014 ke tahun 2016.
(i) State the value of x.
Nyatakan nilai bagi x.
(ii) Find the value of y. [3 marks / markah]
Cari nilai bagi y.
(b) Calculate the composite index for the cost of making the fish balls for the year 2016 based on the year 2014.
Hitung indeks gubahan bagi kos menghasilkan bebola ikan untuk tahun 2016 berdasarkan tahun 2014. [2 marks / markah]
(c) It is given that the composite index for the cost of making the fish balls increased by 40% from the year 2012 to the
year 2016.
Diberi bahawa indeks gubahan untuk kos menghasilkan bebola ikan bertambah sebanyak 40% dari tahun 2012 ke tahun 2016.
(i) Calculate the composite index for the cost of making the fish balls in the year 2014 based on the year 2012.
Hitung indeks gubahan bagi kos menghasilkan bebola ikan dalam tahun 2014 berdasarkan pada tahun 2012.
(ii) The cost of making a fish ball is 10 sen in the year 2012.
Find the maximum number of fish balls that can be produced using an allocation of RM80 in the year 2016.
Kos menghasilkan sebiji bebola ikan ialah 10 sen pada tahun 2012.
Cari bilangan maksimum bebola ikan yang boleh dihasilkan menggunakan peruntukan RM80 dalam tahun 2016.
[5 marks / markah]
Answer Space

114
3 SPM 2016 Paper 2
Table 3 shows the price indices and change in price indices of four raw materials A, B, C and D used to produce a type of
biscuits in a factory.
Jadual 3 menunjukkan indeks harga dan perubahan indeks harga bagi empat jenis bahan mentah A, B, C dan D yang digunakan
dalam menghasilkan sejenis biskut di kilang.
Raw material Price index in year 2011 based on the year 2008 Change in price index from year 2011 to 2015
Bahan mentah Indeks harga dalam tahun 2011 berdasarkan tahun 2008 Perubahan indeks harga dari tahun 2011 ke 2015
A 140 15% increase / bertambah
B 120 5% increase / bertambah
C 160 Unchanged / Tidak berubah
D 150 10% decrease / berkurang
Table 3/ Jadual 3
Diagram 12 is the bar chart which represents the mass of the raw materials used to make the biscuits in 2008.
Rajah 12 ialah carta bar yang menunjukka jisimbahan mentak yang diguakan untuk membuat biskut dalam tahun 2008

Diagram 3 / Rajah 3
(a) The price of raw material A in the year 2011 is RM70.
Find the corresponding price in the year 2008. [2 marks / markah]
Harga untuk bahan mentah A dalam tahun 2011 ialah RM70.
Cari harga yang sepadan dalam tahun 2008.
(b) Find the price indices of all the four raw materials in year 2015 based on the year 2008. [3 marks / markah]
Cari indeks harga untuk semua bahan mentah dalam tahu 2015 berdasarkan pada tahun 2008.
(c) (i) Calculate the composite index for the cost of making the biscuits in year 2015 based on the year 2008.
Hitung indeks gubahan bagi kos menghasilkan biskut dalam tahun 2015 berdaasarkan tahun 2008.
(ii) Hence, find the cost of producing the biscuits in year 2008 if the corresponding cost in 2015 is RM268.20.
Seterusnya, cari kos menghasilkan biskut dalam tahun 2008 jika kos yang sepadan dalam tahun 2015 ialah RM268.20.
[5 marks / markah]
Answer Space

115
4 SPM 2015 Paper 2
Table 4 shows the price indices for the year 2013 and 2015 based on the year 2011 of three materials A, B and C used in
the making of a type of shoe.
Jadual 4 menunjukkan indeks harga untuk tahun 2013 dan 2015 berdasarkan pada tahun 2011 bagi tiga jenis bahan A, B dan C yang
digunakan dalam menghasilkan sejenis kasut.
Material Price index in the year 2013 Price index in the year 2015
Bahan based on the year 2011 based in the year 2011
Indeks harga dalam tahun 2013 Indeks harga dalam tahun 2015
berdasarkan pada tahun 2011 berdasarkan pada tahun 2011
A 106 120
B 105 125
C 110 m
Table 4 / Jadual 4
(a) The price of material C in the year 2011 is RM12.00 and its price in the year 2015 is RM15.60. Find
Harga bagi bahan C dalam tahun 2011 ialah RM12.00 dan harganya dalam tahun 2015 ialah RM15.60. Cari
(i) the value of m,
nilai bagi m,
(ii) the price of material C in the year 2013. [3 marks / markah]
harga bagi bahan C dalam tahun 2013.
(b) The composite index for the production cost of the shoe in the year 2013 based on the year 2011 is 106.7. The
ratio of materials A, B and C used are 2 : h : 3. Find
Indeks gubahan untuk kos penghasilan bagi kasut dalam tahun 2013 berdasarkan pada tahun 2011 ialah 106.7. Nisbah bagi
bahan A, B dan C yang digunakan ialah 2 : h : 3. Cari
(i) the value of h,
nilai bagi h,
(ii) the corresponding price of the shoe in the year 2011 if the price of shoe in the year 2013 is RM58.20 [5]
harga yang sepadan bagi kasut dalam tahun 2011 jika harga kasut dalam tahun 2013 ialah RM58.20.
(c) Find the price index of material B in the year 2015 based on the year 2013. [2 marks / markah]
Cari indeks harga bagi bahan B dalam tahun 2015 berdasarkan pada tahun 2013.
Answer Space

116
5 SPM 2014 Paper 2 Q15
Table 5 shows the price indices and the weightages of four ingredients P, Q, R and S used in the making of a cake. The
composite index for the cost of making the cake in the year 2014 based on the year 2012 is 106.
Jadual 5 menunjukkan indeks harga dan pemberat bagi empat jeis bahan P, Q, R dan S yang digunakan dalam membuat kek. Indeks
gubahan bagi kos menghasilkan kek dalam tahu 2014 berdasarkan tahun 2012 ialah 106.
Ingredient Price index in the year 2014 based on the year 2012 Weightage
Bahan Indeks harga dalam tahun 2014 berdasarkan tahun 2012 Pemberat
P 115 3
Q 95 1
R 100 4
S m 2
Table 5 / Jadual 5
(a) Calculate the price of ingredient Q in the year 2014 if its price in the year 2012 is RM20. [2 marks / markah]
Hitung harga bahan Q dalam tahun 2014 jika harganya dalam tahun 2012 ialah RM20.
(b) Find the percentage of price change from the year 2012 to the year 2014 for ingredient S. [4 marks / markah]
Cari peratus perubahan harga dari tahun 2012 kepada tahun 2014 untuk bahan S.
(c) The composite index for the cost of making the cake increased by 10% from the year 2014 to the year 2015,
calculate
Indeks gubahan bagi kos menghasilkan kek bertambah sebanyak 10% dari tahun 2014 kepada tahun 2015, hitung
(i) the composite index for the expenses in the year 2015 based on the year 2012,
indeks gubahan bagi perbelanjaan dalam tahun 2015 berdasarkan pada tahun 2012,
(ii) the price of the cake in the year 2015 if its corresponding price in the year 2012 is RM75.[4 marks / markah]
harga bagi kek dalam taahun 2015 jika harga yang sepadan dalam tahun 2012 ialah RM75.
Answer Space

117
6 Table 6 shows the price, price indices and the percentage expenditure of four ingredients P, Q, R and S used in the
making of a kind of food.
Jadual 6 menunjukkan harga, indeks harga dan peratus perbelanjaan bagi empat bahan P, Q, R dan S yang digunakan dalam
menghasilkan sejenis makanan.
Ingredient Price (RM) per kg Price index in the year 2007 based on the year 2005 Percentage expenditure (%)
Bahan Harga (RM) per kg Indeks harga dalam tahun 2007 berdasarkan tahun 2005 Peratus perbelanjaan (%)
2005 2007
P 4.00 5.00 x 16
Q 3.00 y 150 12
R 8.00 10.00 125 48
S z 3.00 120 24
Table 6 / Jadual 6
(a) Find the values of x, y and z. [4 marks / markah]
Cari nilai bagi x, y dan z.
(b) Calculate the composite index for the cost of making the food in year 2007 based on the year 2005.
Hitung indeks gubahan bagi kos menghasilkan makanan dalam tahun 2007 berdasarkan tahun 2005. [2 marks / markah]
(c) The cost of making a packet of the food in the year 2005 was RM50.00.
Calculate the corresponding cost in year 2007. [2 marks / markah]
Kos menghasilkan satu pakej makan dalam tahun 2005 ialah RM50.00.
Hitung kos yang sepadan dalam tahun 2007.
(d) The cost of all the ingredients increases by 15% from the year 2007 to the year 2009. Find the composite index for
the year 2009 based on the year 2005. [2 marks / markah]
Kos untuk semua bahan bertambah sebanyak 15% dari tahun 2007 kepada tahun 2009. Cari indeks gubahan bagi tahun 2009
berdasarkan tahun 2005.
Answer Space

118
FORM 4 CHAPTER 10 SOLUTION OF TRIANGLES
PAPER 2 SECTION C
1 Diagram 1 shows a triangle ABD. Point C lies on the straight line BD such that BC = 3.5 cm and AC = AD.
It is given AB = 8 cm and  ABC = 40.
Rajah 1 menunjukkan segi tiga ABD. Titik C berada pada garis lurus BD dengan keadaan BC = 3.5 cm and AC = AD.
Diberi bahawa AB = 8 cm dan  ABC = 40.
A

8 cm

o
40
B D
3.5 cm C
Diagram 1 / Rajah 1
Calculate / Hitung
(a) the length of AD, [3 marks]
panjang AD,
Solution
a b c
Remember the formulae: sine rule = = Length of side must opposite to the angle
sin A sin B sin C

Remember the formulae: cosine rule c2 = a2 + b2 – 2ab cos C Length c must opposite to the angle C
Either one of the above formula will be used to solve the problems involving the length of side or angle!!
Given the length of AD is equal to the length of AC. Use triangle ABC to find the length of AC.
From the cosine rule, c2 = a2 + b2 – 2ab cos C A
AC2 = AB2 + BC2 – 2(AB)(BC) cos B
= 82 + 3.52 – 2(8)(3.5) cos 40º 8 cm
= 33.3515
o
AC = 5.7751 cm 40
B
3.5 cm C
(b)  ACB, [4 marks]
Solution
Use triangle ABC to find the angle of ACB. Remember that the angle of ACB is an obtuse angle!
a b c A
From the sine rule = =
sin A sin B sin C
8 cm
8 5.7751 5.7751 cm
=
sin ACB sin 40 o 40
o
B
Cross multiply: 5.7751 × sinACB = 8 × sin40º 3.5 cm C

8  sin 40 o
sinACB =
5.7751
sinACB = 0.8904
ACB = sin-1 0.8904
= 62.92º
So, the angle of ACB = 180º – 62.92º
= 117.08º
(c) the area of triangle ABD. [3 marks]
luas segi tiga ABD.
Solution
Remember the formulae: Area of triangles, A = ½ a b sin C The angle must between the length of sides used
Since ACB = 117.08º, then ACD = 180º – 117.08º 22.92
o A
= 62.92º
8 cm 54.16 o
Since AC = AD, then ADC = 62.92º 5.7751 cm
So, CAD = 180º – 62.92º – 62.92º o 62.92
o
62.92
o
40
= 54.16º B
3.5 cm C
D

Area of triangle ABD = ½ × AB × AD × sin A


o
[Bolded line] 117.08
= ½ × 8 × 5.7751 × sin (22.92º + 54.16º)
= 22.52 cm2

119
EXERCISE
1 Diagram 1 shows a trapezium ABCD in which AB and DC are parallel.
Rajah 1 menunjukkan trapezium ABCD dengan keadaan AB dan DC adalah selari.
9.8 cm
D C
118 o
6.4 cm

A B
5 cm

Diagram 1 / Rajah 1
Given that AB = 5 cm, AD = 6.4 cm, DC = 9.8 cm and  ADC = 118.
Diberi bahawa AB = 5 cm, AD = 6.4 cm, DC = 9.8 cm dan  ADC = 118.
Find / Cari
(a) the length, in cm, for / panjang, dalam cm, bagi
(i) AC, [2 marks] [Ans: 13.9961]

(ii) BD, [2 marks] [Ans: 5.9928]

(b)  ABD, [3 marks] [Ans: 70.54º]

(c) the area, in cm2, of triangle BCD. [3 marks] [Ans: 27.6872]


luas, dalam cm2, segi tiga BCD.
Answer space

120
2 Diagram 2 shows a quadrilateral ABCD where  ABC is an acute angle.
Rajah 2 menunjukkan segi empat ABCD dengan keadaan  ABC ialah sudut tirus.

D
5.2 cm
C
9.6 cm 12.6 cm 8.7 cm

o
A 38.5 B
Diagram 2 / Rajah 2
(a) Calculate / Hitung
(i)  ABC, [2 marks] [Ans: 64.37º]

(ii)  ADC, [2 marks] [Ans: 113.34º]


2
(iii) the area, in cm , of quadrilateral ABCD. [3 marks] [Ans: 76.3506]
luas, dalam cm2, segi empat ABCD.
(b) A triangle ABC has the same measurements as those given for triangle ABC, that is, AC = 12.6 cm, CB = 8.7
cm and  CAB = 38.5 but has the different shape compared with the triangle ABC where AC = 12.6 cm.
Segi tiga ABC mempunyai ukuran yang sama dengan segi tiga ABC, di mana, AC = 12.6 cm, CB = 8.7 cm dan  CAB =
38.5 tetapi mempunyai bentuk yang berbeza berbanding dengan segi tiga ABC dengan keadaan A C = 12.6 cm.
(i) Sketch the triangle ABC. [1 mark]
Lakarkan segitiga ABC.
(ii) Calculate the length, in cm, of AB. [2 marks] [Ans: 6.098]
Hitung panjang, dalam cm, AB.
Answer space

121
3 Diagram 3 shows a pyramid PQRS with triangle PQR as the horizontal base.
Rajah 3 menunjukkan piramid PQRS dengan segitiga PQR sebagai satah mengufuk.
S

R P

Q
Diagram 3 / Rajah 3
S is the vertex of the pyramid and the angle between the inclined plane QRS and the base PQR is 50º. Given that PQ =
PR = 5.6 cm and SQ = SR = 4.2 cm.
S ialah puncak bagi piramid itu dan sudut antara satah condong QRS dengan dasar PQR ialah 50. Diberi PQ = PR = 5.6 cm dan
SQ = SR = 4.2 cm.
Calculate / Hitungkan
(a) the length of RQ if the area of the base PQR is 12.4 cm2, [3 marks] [Ans: 4.9326]
panjang RQ jika luas dasar PQR ialah 12.4 cm2,
(b) the length of SP, [3 marks] [Ans: 3.8551]
panjang SP,
(c) the area of the triangle PQS. [4 marks] [Ans: 8.0982]
luas segitiga PQS.
Answer space

122
INTENSIVE EXERCISE
FORM 4 CHAPTER 10 SOLUTION OF TRIANGLES
1 Diagram 1 shows the triangle ABC and triangle CDE where ACE and BCD are straight lines.
Rajah 1 menunjukkan segitiga ABC dan segitiga CDE dengan keadaan ACE dan BCD adalah garis lurus.

Diagram 1 / Rajah 1
(a) Calculate the length, in cm, for
Hitung panjang, dalam cm, untuk
(i) AC,
(ii) DE. [5 marks / markah]
(b) Point C’ is on AE such that BC’ = BC.
Titik C’ terletak pada AE dengan keadaan BC’ = BC.
(i) Sketch the triangle BC’A.
Lakar segitiga BC’A.
(ii) Find the BC’A.
Cari BC’A.
(iii) Calculate the area, in cm2, for triangle BC’C. [5 marks / markah]
Hitung luas, dalam cm2, untuk segitiga BC’C.
Answer Space

123
2 Diagram 2 shows a triangles ABC.
Rajah 2 menunjukkan suatu seitiga ABC.

Diagram 2 / Rajah 2
(a) Calculate the length of AC, in cm. [2 marks / markah]
Hitung panjang AC, dalam cm.
(b) Quadrilateral ABCD is formed such that AC is the diagonal, ACD = 30º and AD = 12 cm.
Calculate the possible values of ADC. [2 marks / markah]
Segiempat ABCD terbentuk dengan keadaan AC ialah pepenjuru, ACD = 30º dan AD = 12 cm.
Hitung nilai-nilai yang mungkin bagi ADC.
(c) Using the acute angle for ADC in 1(b), calculate
Menggunakan sudut tirus untuk ADC dalam 1(b), hitung
(i) length of CD in cm, [3 marks / markah]
panjang CD dalam cm,
(ii) area of ABCD, in cm2. [3 marks / markah]
luas bagi ABCD, dalam cm2.
Answer Space

124
3 Solution by scale drawing is not accepted.
Penyelesaian secara lukisan berskala tidak diterima.
Diagram 3 shows a transparent prism with a rectangular base ABCD. The inclined surface ABFE is a square
with sides 12 cm and the inclined surface surface CDEF is a rectangle. AED is a uniform cross section of the
prism. BDE is a blue coloured plane in the prism.
Rajah 3 menunjukkan prisma lutsinar dengan tapak ABCD berbentuk segiempat tepat. Permukaan condong ABFE ialah
segi empat dengan sisi 12 cm dan permukaan condong CDEF ialah segi empat tepat. AED ialah keratan rentas seragam
bagi prisma itu. BDE ialah satah berwarna biru di dalam prisma itu.

Diagram 3 / Rajah 3
It is given that ADE = 37º and EAD = 45º.
Diberi bahawa ADE = 37º dan EAD = 45º.
Find / Cari
(a) the length, in cm, of DE, [2 marks / markah]
panjang, dalam cm, bagi DE,
(b) the area, in cm2, of the blue coloured plane, [6 marks / markah]
luas, dalam cm2, satah berwarna biru,
(c) the shortest length, in cm, from point E to the straight line BD. [2 marks / markah]
panjang terdekat, dalam cm, dari titik E ke garis lurus BD.
Answer Space

125
4 Diagram 4 shows a triangle PQR. The point T lies on the straight line PR. The area of triangle QTR is 17.5
cm2 and QTR is obtuse.
Rajah 4 menunjukkan suatu segitiga PQR. Titik T terletak pada garis lurus PR. Luas segitiga QTR ialah 17.5 cm2 dan
QTR ialah cakah.

Diagram 2 / Rajah 2
Calculate / Hitung
(a) QTR [3 marks / markah]
(b) the length, in cm, of QR, [2 marks / markah]
panjang, dalam cm, bagi QR,
(c) QPT [2 marks / markah]
2
(d) the area, in cm , of triangle PQR. [3 marks / markah]
luas, dalam cm2, bagi segitiga PQR.
Answer Space

(a) 150° (b) 16.4391 cm (c) 23°9’ (d) 42.4265 cm2

126
5 Diagram 5 shows a trapezium PQRS. QP is parallel to RS and RSP is an obtuse angle.
Rajah 5 menunjukkan suatu trapezium PQRS. QP adalah parallel dengan RS dan RSP ialah sudut cakah.

Diagram 5 / Rajah 5
Find / Cari
(a) the length, in cm, of PR, [2 marks / markah]
panjang, dalam cm, bagi PR,
(b) the length, in cm, of PS, [3 marks / markah]
panjang, dalam cm, bagi PS,
(c) RSP, [3 marks / markah]
(d) the area, in cm2, of triangle PRS. [2 marks / markah]
luas, dalam cm2, bagi segitiga PRS.
Answer Space

END OF MODULE

127

Você também pode gostar